Pediatrics Exam 1

Lakukan tugas rumah & ujian kamu dengan baik sekarang menggunakan Quizwiz!

Which would be an acceptable community-health diagnosis? 1. Risk for Injury Related to Lack of Safe Bicycle Paths in High-Traffic Areas 2. Ineffective Family Coping Related to Lack of Time Together 3. Alterations in Nutrition Related to Use of Fast Food Restaurants 4. Ineffective Communication Related to Lack of Community Newsletter

Answer: 1 Explanation: 1. The lack of safe bicycle paths in high-traffic areas is a community hazard affecting a large population of people. Ineffective family coping is appropriate for one family; alterations in nutrition and ineffective communication are not appropriate for the community as a whole. Page Ref: 209, 210

The nurse is working with a school-age child who is hospitalized. Which action by the nurse will promote a sense of industry in this child? 1. Allow the child to assist with her care. 2. Encourage parents to participate in the child's care. 3. Give the child a detailed scientific explanation of the illness. 4. Speak to the child in a high-pitched voice.

Answer: 1 Explanation: Allowing the child to participate in her care will decrease the sense of loss of control and increase a sense of industry. While parents can certainly participate in their child's care, it does not increase the child's sense of control. School-age children in general will not understand detailed scientific explanations. Change in voice tone is appropriate when talking to very young children. Page Ref: 225-227

An adolescent client has a long leg cast secondary to a fractured femur. Which action by the nurse would effectively facilitate the adolescent's return to school? 1. Meet with teachers and administrators at the school to make sure entrances and classrooms are wheelchair accessible. 2. Develop an individualized health plan (IHP) that focuses on long-term needs of the adolescent. 3. Prior to the student's return to school, meet with all of the other students to emphasize the special needs of the injured teen. 4. Meet with parents of the injured student to encourage homebound schooling until a short leg cast is applied.

Answer: 1 Explanation: An adolescent with a long leg cast secondary to a fractured femur will be dependent on a wheelchair for mobility. It is essential that the environment be wheelchair accessible prior to the adolescent's return to school. While an IHP might be developed, short-term needs would be the focus. It is not necessary to meet all of the students to discuss the adolescent's needs. There is no reason to encourage the adolescent to stay at home for schooling if he is ready to return. Page Ref: 212

A school-age client is admitted to the pediatric intensive care unit (PICU) in critical condition after a motor vehicle accident. Which intervention should be implemented at this time? 1. Maintain consistent caregivers. 2. Turn the lights off at night. 3. Keep alarm levels low. 4. Consult the hospital play therapist.

Answer: 1 Explanation: The intensive care environment is fast-paced, overwhelming, and frightening. Maintaining consistent caregivers is invaluable in developing a familiar and trusting relationship with the child. Turning off the lights in an intensive care environment is not feasible. Keeping alarm levels low could increase risk of injury if an alarm is not heard by staff. Consulting the play therapist is not appropriate at this time. Page Ref: 268

The nurse is providing care to an adolescent client who is dying. Which assessment findings indicate the client is experiencing a decrease in peripheral circulation? Select all that apply. 1. Cool skin 2. Mottled appearance 3. Cheyne-Stokes respirations 4. Increased agitation 5. Increased urine output

Answer: 1, 2 Explanation: A client who is experiencing decreased peripheral circulation will have cool, mottled skin. While Cheyne-Stokes respirations may indicate death is approaching, this is not indicative of a decrease in peripheral circulation. Increased agitation indicates decreased perfusion to the brain. A client will not experience increased urine output near the end life. Page Ref: 273

The school nurse performs screenings on all students in the middle school. In addition, the nurse will perform selected screenings on individual school-age children. When planning the screenings for the year, which screenings will the nurse include for all school-age children? Select all that apply. 1. Hearing 2. Height and weight 3. Blood-pressure measurement 4. Hepatitis B profile serology 5. Chest x-ray

Answer: 1, 2, 3 Explanation: Routine screening for school-age children include hearing, checking for height and weight, and blood-pressure measurements. The hepatitis B profile is only needed once, prior to administration of hepatitis B vaccine; however, this is not a required screening for all school-age children. A chest x-ray is not a routine screening test for school-age children. Page Ref: 185

The nurse is conducting a nutritional assessment for a toddler client who is diagnosed with failure to thrive (FTT). Which parameters will the nurse include in the assessment process for this toddler and family? Select all that apply. 1. Height 2. Weight 3. Hemoglobin and hematocrit 4. Twenty-four-hour food diary 5. Maternal dietary intake during pregnancy

Answer: 1, 2, 3, 4 Explanation: In order to adequately assess the toddler client's FTT, the nurse would plan to measure height and weight; obtain a hemoglobin and hematocrit; and ask the family for a 24-hour food diary. Information regarding maternal dietary intake during pregnancy is not information that is necessary to assess for a toddler diagnosed with FTT. Page Ref: 300

An adolescent is accompanied by the mother for an annual physical examination. The nurse is aware of privacy issues related to the adolescent. While the mother is in the room, the nurse should avoid which questions? Select all that apply. 1. Sexual activity 2. Cigarette smoking 3. School performance 4. Use of alcohol 5. Car seatbelt use

Answer: 1, 2, 4 Explanation: The nurse must maintain the nurse—client relationship, which is between the nurse and the adolescent, and the nurse must maintain confidentiality. Therefore, the nurse cannot ask any personal questions while the mother is in the room, such as those related to sexual activity, drug and alcohol use, and smoking cigarettes. The nurse can ask general questions about seatbelt use and academic performance without breaching confidentiality. Page Ref: 184

When reviewing the adolescent health record, which immunizations should the nurse encourage? Select all that apply. 1. Varicella 2. Human papillomavirus 3. HIV 4. Cholesterol 5. Hepatitis B

Answer: 1, 2, 5 Explanation: When identifying immunizations needed by the adolescent some of the questions to ask would be: When was the last tetanus-diphtheria (Td) booster? Was a second measles-mumps-rubella administered? Is hepatitis A common in your state? Has the youth had hepatitis B vaccine? Did the youth have a documented history of varicella disease? Has the youth received meningococcal vaccine? Have the adolescent female and male received the human papillomavirus vaccine? Has the youth received the annual influenza vaccine? Page Ref: 203

The nurse is teaching the adolescent and family about sleep hygiene. What behaviors should the nurse suggest? Select all that apply. 1. Avoid naps in the late afternoon and evening 2. Sleep 12 hours a day 3. Avoid caffeine, tea, coffee, carbonated beverages and energy drinks for several hours before sleep. 4. Avoid setting an alarm clock 5. Go to bed and get up at the same time each day, including weekends

Answer: 1, 3, 5 Explanation: 1. General information about sleep includes no drinks or food with stimulants, go to bed and get up at the same time each day, including weekends, and avoid naps in the late afternoon and evening. Page Ref: 189

A nurse is assessing language development in all the infants presenting at the doctor's office for well-child visits. At which age range would the nurse expect a child to verbalize the words "dada" and "mama"? 1. 3 and 5 months 2. 6 and 8 months 3. 9 and 12 months 4. 13 and 18 months

Answer: 3 Explanation: Children should be able to verbalize "mama" or "dada" to identify their parents by 1 year of age. Page Ref: 87

The school health nurse is evaluating the home environment of several children as it relates to child safety. The nurse visits the home of each child and gathers the following data. Which activity places a child at greatest risk for bodily harm? 1. The parents are in a methadone program. 2. The parents consume alcohol on a daily basis. 3. The child is permitted to target practice with a revolver, unsupervised. 4. The child is a latchkey child.

Answer: 3 Explanation: Of all the activities mentioned, the child who is playing with guns is most at risk for injury. The inappropriate behaviors, such as drug and alcohol use or past use, also place the child at risk, but the use of firearms is more risky. A latchkey child needs special attention but in regard to the situations given is not at the greatest risk of injury. Page Ref: 190, 191

The nurse is assessing a 14-year-old and notes signs and symptoms of bulimia nervosa. Which assessments led the nurse to this conclusion? Select all that apply. 1. Pale skin 2. Dry, splitting hair 3. Erosion of tooth enamel 4. Calluses on back of hand 5. Gum recession

Answer: 3, 4, 5 Page Ref: 303-304

The nurse is caring for a toddler client in the postoperative period. Which pain assessment tool is most appropriate for this client? 1. FLACC Behavioral Pain Assessment Scale 2. FACES pain scale 3. Oucher scale 4. Poker-chip tool

Answer: 1 Explanation: 1. The FLACC scale is an appropriate tool for infants and young children who cannot self-report pain. The FACES Scale, Oucher scale, and poker-chip tool are all self-report scales. Page Ref: 316-318

An adolescent reports the following: "I get up at 6 a.m., I attend early-morning band classes three times each week, I play sports for 2 hours each day after school, and homework takes me 3 hours each night. I always feel tired." Which question by the nurse is most appropriate based on this information? 1. "How many hours of sleep do you get each night?" 2. "Do you consume foods high in iron?" 3. "Do you think you are doing too much?" 4. "Have you considered talking with your teachers about decreasing your homework, since you have so many extracurricular activities?"

Answer: 1 Explanation: 1. The data in this scenario reveals very little time for sleep; therefore, the history should focus on sleep patterns. Page Ref: 185, 187

The school nurse completes an assessment of a school-age client to determine the services this child will need in the classroom. The client is a newly diagnosed with type I diabetes mellitus. Based on this information, which special healthcare need category is the most appropriate? 1. Dependent on medication or special diet 2. Dependent on medical technology 3. Increase use of healthcare services 4. Functional limitations

Answer: 1 Explanation: A child recently diagnosed with type I diabetes mellitus with no other medical diagnoses would be placed in the dependent on medication or special diet category. The other categories of care are not appropriate for this client. Page Ref: 248

A toddler-age client is in end-stage renal failure. Which nursing intervention will assist this child most? 1. Maintain the child's normal routines. 2. Explain body changes that will take place. 3. Encourage friends to visit. 4. Allow the child to talk about the illness.

Answer: 1 Explanation: A toddler has no real concept of death, but does sense changes in routine and parent behavior. Maintaining normal routines is the best intervention to assist this child. A toddler will not understand the body changes; this approach would be more appropriate for a school-age child. Encouraging friends to visit and allowing the child to talk about the illness are more appropriate for older children. Page Ref: 274-275

The nurse is working with an adolescent client who will be admitted to the hospital in two days. Which nursing approach is most appropriate to prepare this client for hospitalization? 1. Have teens who have had similar experiences talk to the adolescent about hospitalization. 2. Provide an opportunity for the child to talk with an adult who has had a similar experience. 3. Teach parents what to expect so the information can be shared with the adolescent. 4. Provide an opportunity for the teen to try on surgical attire.

Answer: 1 Explanation: Adolescents benefit from a different approach than younger children when being prepared for hospitalization. Written materials, anatomically correct dolls, and talking to peers who have had similar experiences are all appropriate for the adolescent. The adolescent should be taught first-hand what to expect during the hospitalization. Dressing up in surgical attire is appropriate for the younger child. Page Ref: 233

When examining a 7-year-old, which action by the nurse would be most appropriate? 1. Allow the child to participate in the exam. 2. Ask the parent what kind of food the child likes to eat. 3. Ask the child whether he plays outside for at least 30 minutes a day. 4. Allow the child to decide whether he is ready for his next immunization.

Answer: 1 Explanation: At this age, children have logical thought, and are learning about their bodies. Participating in the physical exam is appropriate for this age. The child can answer the question about food intake himself. Asking whether he plays outside for 30 minutes is fine, but children at this age need at least 60 minutes of activity, so the question will not gather appropriate information. It is not the child's decision whether he is ready for immunization, so do not ask this question. Page Ref: 191, 192

There are many healthcare needs of children with chronic conditions. What nursing strategy would best help parents with continuity of care? 1. Include the family and older child in decision making. 2. Assist the family in gaining transportation to healthcare appointments. 3. Provide the family with resources such as social services. 4. Recognize and respect the cultural needs of the family.

Answer: 1 Explanation: Continuity of care involves the family and child's participation in their health care. Access to transportation involves access to care, not continuity. Providing resources such as social services is related to comprehensiveness of care, not to continuity. Recognizing and respecting cultural needs are part of the degree to which healthcare services, not continuity of care, are provided. Page Ref: 249

While teaching the parents of a newborn about infant care and feeding, which instruction by the nurse is the most appropriate? 1. Delay supplemental foods until the infant is 4 to 6 months old. 2. Delay supplemental foods until the infant reaches 15 pounds or greater. 3. Begin diluted fruit juice at 2 months of age, but wait 3 to 5 days before trying a new food. 4. Add rice cereal to the nighttime feeding if the infant is having difficulty sleeping after 2 months of age.

Answer: 1 Explanation: Four to six months is the optimal age to begin supplemental feedings because earlier feeding of nonformula foods is not needed by the infant and does not promote sleep. Earlier feeding of nonformula foods, regardless of the infant's weight, is more likely to cause the development of food allergies. Also, early feeding is not well tolerated by infants because the necessary tongue control is not well developed and they lack the digestive enzymes to take in and metabolize many food products. Page Ref: 285-286

The nurse working in the clinic includes an adolescent history in every client intake interview. Which issue should the nurse address when the parents are not present? 1. Possible domestic violence 2. Teen job responsibilities 3. Activities that are done as a family 4. The adolescent's role in the family

Answer: 1 Explanation: If domestic violence is suspected, it would only be appropriate to ask these questions when the teenager is alone with the nurse or healthcare provider. Page Ref: 188, 189

A young school-age client is hospitalized with a fractured femur. Which assessment tools are appropriate for this client? Select all that apply. 1. FACES pain scale 2. Oucher scale 3. Visual Analog Scale 4. CRIES Scale 5. Poker-chip tool

Answer: 1, 2, 5 Explanation: A young school-age client should be able to use the FACES Scale and Oucher scale to choose which face best matches the child's pain level. The child should also be able to count and understand the concepts of the poker-chip tool. The CRIES Scale was developed for preterm and full-term neonates. A young school-age client is not old enough to use the Visual Analog Scale. Page Ref: 316-318

The nurse is teaching the parents of a 4-month-old infant about good feeding habits. The nurse emphasizes the importance of holding the baby during feeding and not letting the infant go to sleep with the bottle. Which disorder is associated with propped feedings and going to sleep with the bottle? 1. Otitis media 2. Aspiration 3. Malocclusion problems 4. Sleeping disorders

Answer: 1 Explanation: It has been shown in numerous studies that allowing an infant to fall asleep with a bottle in his or her mouth causes pooling of the formula in the mouth, which increases the risk of both dental caries and otitis media. There has been limited data to date showing a positive correlation between bottle propping and increased risk of aspiration, malocclusions, and sleeping disorders. Page Ref: 284

The nurse is working with a preschool-age client in Bryant traction for a fractured femur. Why is the Oucher Scale useful to the nurse caring for this child? 1. It provides continuity and consistency in assessing and monitoring the child's pain. 2. It decreases anxiety in the child. 3. It increases the child's comfort level. 4. It reduces the child's fear of painful procedures.

Answer: 1 Explanation: Pain assessment scales are used to assess and monitor pain. Using an assessment scale cannot reduce the child's anxiety or fear, nor can it increase the child's comfort level. The nurse can reduce anxiety or fear and increase the child's comfort level by implementing appropriate nursing interventions based on assessment scale data. Page Ref: 317

The nurse is working with a child with a chronic condition. The nurse observes that over time, the parents have experienced a pattern of periodic grieving alternating with denial. What are the parents currently experiencing based on this assessment finding? 1. Chronic sorrow 2. Compassion fatigue 3. Dysfunctional parenting 4. Pathological grieving

Answer: 1 Explanation: Parents experience chronic sorrow as they grieve when their child does not meet developmental milestones or participate in activities of "normal" children. The time between periods of grieving may be times of parental denial, which allows the family to function. Compassion fatigue is experienced by caregivers as their ability to feel compassion is exhausted. Dysfunctional parenting involves inadequately meeting the needs of children. Pathological grieving results when persons do not move through the stages of grief to resolution. Page Ref: 258

A toddler is hospitalized with a fractured femur. In addition to pain medication, which will best provide pain relief for this child? 1. Parents' presence at the bedside 2. Age-appropriate toys 3. Deep-breathing exercises 4. Videos for the child to watch

Answer: 1 Explanation: Parents' presence at the bedside reduces anxiety and subsequently reduces pain. Although play and other methods of distraction might be somewhat effective, they do not equal the comfort that parents' presence provides, especially in a 2-year-old, who is also at high risk for separation anxiety. Page Ref: 327

A child who is dependent on a ventilator is being discharged from the hospital. Prior to discharge, the home-health nurse discusses development of an emergency plan of care with the family. Which is the most essential part of the plan? 1. Acquisition of a backup generator 2. Designation of an emergency shelter site 3. Provision for an alternate heating source if power is lost 4. Notifying the power company that the child is on life support

Answer: 1 Explanation: Prior to discharge to home, it is essential that the family acquire a generator so that the child's life support will continue to function effectively should power be lost. While all other actions are very important, it is most essential that the ventilator has power to continue to function at all times. Page Ref: 214

During a 4-month-old's well-child check, the nurse discusses introduction of solid foods into the infant's diet and concerns for foods commonly associated with food allergies. Due to allergies, which foods will the nurse instruction the parents to avoid until after 1 year of age? 1. Strawberries, eggs, and wheat 2. Peas, tomatoes, and spinach 3. Carrots, beets, and spinach 4. Squash, pork, and tomatoes

Answer: 1 Explanation: Strawberries, eggs, and wheat, along with corn, fish, and nut products, are all foods that have commonly been associated with food allergies. Carrots, beets, and spinach contain nitrates and should not be given before the age of 4 months. Squash, peas, and tomatoes are acceptable to try after an infant is 4 to 6 months old but should be given one at a time and 3 to 5 days after starting a new food. Pork can be tried after the infant is 8 to 10 months old, as meats are harder to digest and have a high protein load. Page Ref: 285-287

Parents of a child who will begin enteral feedings ask the nurse what advantage this type of feeding has over other methods. Which responses by the nurse are the most appropriate? Select all that apply. 1. "Enteral feeding is the closest to natural feeding methods." 2. "The child must be able to absorb nutrients." 3. "Enteral feeding is complex to administer." 4. "Enteral feeding requires a central venous catheter." 5. "Enteral feeding has a high success rate."

Answer: 1, 2, 5 Explanation: Enteral feedings are the closest to natural feeding methods. The child must be able to absorb nutrients. Enteral feeding has a high success rate. It is not complex to administer, and does not require a central venous catheter. Page Ref: 307-308

A 5-year-old sibling of a 9-year-old child with cystic fibrosis tells the nurse, "I wish I had a breathing disease, too." The nurse knows the parents strive to spend quality time with each child and with both children together. What is the sibling currently experiencing? 1. Jealousy 2. Isolation 3. Loneliness 4. Anger

Answer: 1 Explanation: The child with cystic fibrosis has something the younger child does not have. Cystic fibrosis brings the affected child more attention from others. Even if parents strive to spend more time with siblings of ill children, the well-child will be jealous because the situation can never be equal. The 5-year-old child does not understand the complications of the disease and only sees the 9-year-old child treated differently. Siblings of ill children may experience loneliness, isolation, or anger; but the child's comment does not support these feelings. Page Ref: 248-249

The nurse prepares a DTaP (diphtheria, tetanus toxoid, and acellular pertussis) immunization for a 6-month-old infant. To administer this injection safely, the nurse chooses which needle, size and length, injection type, and injection site? 1. 25-gauge, 5/8-inch needle; IM (intramuscular); anterolateral thigh. 2. 22-gauge, 1/2-inch needle; IM (intramuscular); ventrogluteal. 3. 25-gauge, 5/8-inch needle; ID (intradermal); deltoid. 4. 25-gauge, 3/4-inch needle; SQ (subcutaneous); anterolateral thigh.

Answer: 1 Explanation: The dose of DTaP is 0.5 cc or 0.5 mL, to be given with a 22 to 25-gauge, 5/8- to 3/4-inch needle; IM (intramuscularly). The only safe intramuscular injection site for a 6-month-old infant is the anterolateral thigh. Page Ref: 348

The nurse is working with a hospitalized preschool-age child. The nurse is planning activities to reduce anxiety in this child. Which action by the nurse is the most appropriate? 1. Provide the child with a doll and safe medical equipment. 2. Read a story to the child. 3. Use an anatomically correct doll to teach the child about the illness. 4. Talk to the child about the hospitalization.

Answer: 1 Explanation: Therapeutic play is a means of anxiety reduction in the hospitalized child. Allowing the child to play with safe medical equipment is an age-appropriate method through which the child can express her feelings, thereby reducing anxiety. Anatomically correct dolls are not age appropriate. Reading a story to the child does not allow for expression of feelings. Talking to the child may be beneficial, but it does not allow for active release of frustration and anxiety as active play does. Page Ref: 235

The nurse is planning activities for a toddler with a birth injury of a torn brachial plexus that resulted in muscle atrophy and weakness of his right arm. Which nursing intervention is most appropriate for this client? 1. Offering the toddler a choice of clothing 2. Asking the toddler if he would like to take his medicine 3. Dressing the toddler 4. Feeding the toddler

Answer: 1 Explanation: Toddlers are developing autonomy, self-control, and independence. Offering the toddler a choice contributes to their sense of autonomy. However, taking medicine is not within the toddler's realm of choice. Dressing and feeding the toddler does not encourage independence and will eventually cause frustration for both parent and toddler. The toddler must learn how to do these activities despite the physical limitations of the right arm. Page Ref: 252

A school-age child has been seen in the pediatric clinic three times in the last two months for complaints of abdominal pain. Physical exam and all ordered lab work have been normal. Which question by the nurse would most likely help determine the etiology of the child's abdominal pain? 1. "Have there been any changes in your child's school or home life recently?" 2. "How many meals does your child eat each day?" 3. "Are your child's immunizations up to date?" 4. "Has your child had any fevers or viral illnesses in the last three months?"

Answer: 1 Explanation: With a normal exam and lab work there is a high probability that this child's abdominal pain is stress related, and it is most important to identify the possible stressors in this child's life to aid in diagnosis and treatment. The other questions are also important to ask but are not as relevant to this child's symptoms as "Have there been any changes in your child's school or home life recently?" Page Ref: 371

During a well-child exam, the parents of a preschool-age child inform the nurse that they are thinking of buying a television for their child's bedroom and ask for advice as to whether this is appropriate. Which response by the nurse is the most appropriate? 1. "Research has shown that children with a television in their bedroom spend significantly less time playing outside than other children, and physical inactivity in children has been linked to many chronic diseases such as obesity and type 2 diabetes." 2. "Research has shown that watching educational television shows improves a child's performance in school." 3. "Don't buy a television for your child's room; he is much too young for that." 4. "It is okay for children to have a television in their room as long as you limit the amount of time they watch it to less than two hours per day."

Answer: 1 Explanation: Young children need to be physically active at this age. "Research has shown that children with a television in their bedroom spend significantly less time playing outside than other children, and physical inactivity in children has been linked to many chronic diseases such as obesity and type 2 diabetes" is the best response because it gives the parents an evidence-based rationale for not placing a television in the child's room. "Don't buy a television for your child's room; he is much too young for that" does not give parents a rationale and may seem opinionated to them. While there may be some truth in the comment "Research has shown that watching educational television shows improves a child's performance in school," this statement may encourage increased television watching by the child, and the child's developmental need for physical activity is greater than the benefit that he may obtain by watching educational programs. "It is okay for children to have a television in their room as long as you limit the amount of time they watch it to less than two hours per day" is correct in that limiting television viewing to less than two hours per day is appropriate, but the probability of this occurring with a television in the child's room is low, and the child will most likely be watching much more than two hours per day. Page Ref: 381

During the nurse's initial assessment of a school-age child, the child reports a pain level of 6 out of 10. The child is lying quietly in bed watching television. Which action by the nurse is most appropriate? 1. Administer prescribed analgesic. 2. Ask the child's parents if they think the child is hurting. 3. Reassess the child in 15 minutes to see if the pain rating has changed. 4. Do nothing, since the child appears to be resting.

Answer: 1 Explanation: School-age children are old enough to accurately report their pain level. A pain score of 6 is an indication for prompt administration of pain medication. The child may be trying to be brave or may be lying still because movement is painful. Page Ref: 313

Parents of a child in the pediatric intensive care unit (PICU) have been experiencing shock and disbelief regarding their situation. Which statement by the parents indicates they are moving forward into the next stage of coping? 1. "Why not me instead of my child?" 2. "It is hard for me to have others take care of my child." 3. "I feel like life is suspended in time." 4. "I am glad I can help with his care."

Answer: 1 Explanation: The parents initially enter the stage of shock and disbelief. Asking "Why not me instead of my child?" shows they are moving into the next stage, which is anger and disbelief. Having feelings about others caring for their child is the third stage of deprivation and loss. The feeling of being suspended in time is the fourth stage, which is anticipatory guidance. Page Ref: 266

The school nurse is planning a smoking-prevention program for middle school students. Which intervention is most likely to be effective in preventing middle school children from smoking? 1. Having a local high school basketball star come to talk to the students about the importance of not smoking 2. Having the school's biology teacher demonstrate the pathophysiology of the effects of smoking tobacco on the body 3. Developing colorful posters with catchy slogans and placing them all over the school 4. Having a pledge campaign with prizes awarded, during which students sign contracts saying that they will not use tobacco products

Answer: 1 Explanation: While all of the strategies are good, the most effective tip would be to have a local high school basketball star come to talk to the students about the importance of not smoking, because students at this age are more likely to listen to and attempt to emulate someone of their own peer group whom they respect and look up to. Information from adults, posters, and signed contracts are not as likely to influence children of this age more than the pressure of their peers. Page Ref: 375—376

The nurse teaches a group of parents' strategies to reduce the risk of lead exposure for their children. Which statements indicate an appropriate understanding of the content presented? Select all that apply. 1. "We will provide our child with frequent snacks high in iron and calcium." 2. "We will wash any surfaces that have peeling paint." 3. "We will store leftovers in a ceramic pot." 4. "We can continue to use our traditional-medicine treatment, Azarcon, for any GI upset." 5. "We will sand the windowsills to remove the lead-based paint."

Answer: 1, 2 Explanation: Snacks and meals high in iron and calcium should be encouraged. Lead is absorbed more readily on an empty stomach. Any surface with peeling paint should be washed with a damp sponge. Ceramic pots, if fired improperly, could contain lead. Food should not be prepared or stored in them. Azarcon, a traditional medicine used to treat a colic-like illness, may contain large amounts of lead. Sanding the windowsills will cause the lead to be dispersed in the air, leading to lead poisoning. Page Ref: 400-401

The school nurse is implementing a program to decrease bullying. Which interventions are appropriate for the school nurse to implement? Select all that apply. 1. Train teachers about the behaviors 2. Ensure adult supervision in the hallways 3. Teach children to report behaviors 4. Ensure that immunizations are up-to-date 5. Set up anti-hazing policies

Answer: 1, 2, 3 Explanation: Appropriate interventions for the school nurse to implement when dealing with bullying in a school include training the teachers on the signs of bullying; ensuring adult supervision in the hallways, as this is where bullying tends to take place; and teach children to report bullying behaviors. Ensuring that immunizations are up to date is not an intervention aimed at decreasing bullying. Hazing and bullying are two separate problems. Page Ref: 385-386

A novice nurse in the newborn intensive care unit (NICU) has just performed postmortem care on a premature infant who passed away. The novice nurse asks to be excused near the end of the shift. Which interventions can be implemented to support this nurse? Select all that apply. 1. Schedule additional education on bereavement care 2. Ask a seasoned nurse to talk with the novice nurse 3. Tell the nurse it is OK to grieve with the family 4. Recommend that the nurse transfer to another unit 5. Assign the nurse to stable clients only

Answer: 1, 2, 3 Explanation: Appropriate interventions for this nurse include scheduling additional education on bereavement care, asking a seasoned nurse to talk about the situation with the novice nurse, and telling the nurse it is OK to grieve with the family. Recommending a transfer and assigning the nurse to only stable clients are not appropriate interventions to support the novice nurse. Page Ref: 279

The school health nurse recognizes that children who display certain characteristics are at risk for poor school performance. The nurse will, therefore, observe each school-age child for which characteristics? Select all that apply. 1. Decreased ability to perform visual tracking. 2. Decreased auditory stimulation. 3. Decreased muscle tone. 4. Multiple dental caries. 5. Chronic tonsillitis.

Answer: 1, 2, 3 Explanation: Children with vision, hearing, and muscle tone problems are at risk for poor school performance, since most school activities involve listening, seeing, and kinetic activity. School performance most likely would not be affected by dental caries and chronic tonsillitis. Page Ref: 189, 190

The nurse is planning care for a preschool-age child and family. In order to assess the family, what should the nurse plan to do during each health supervision visit? Select all that apply. 1. Discuss the child's developmental status 2. Observe interactions among the family members 3. Discuss concerns with the parents 4. Administer age appropriate vaccinations 5. Record height and weight

Answer: 1, 2, 3 Explanation: In order to assess the child and family, the nurse would plan to discuss the child's developmental status, observe interactions among the family members, and discuss any concerns with the parents. Administering age appropriate vaccinations and recording height and weight are appropriate interventions, but are not included during the family assessment process. Page Ref: 168, 169

The nurse is conducting an admission assessment for a preschool-age client in the emergency department. When using the resiliency theory, which findings place this client at risk? Select all that apply. 1. Loss of health insurance 2. No primary care provider 3. Incomplete immunizations 4. A grandmother who is able to room-in 5. High level language skills from the child

Answer: 1, 2, 3 Explanation: When using the resiliency theory, a child and family will have both protective and risk factors. Risk factors include lack of health insurance, not having a consistent care provider, and incomplete immunizations. Protective factors include a parent or family member being able to room-in with the child, a family who is able to stay with the other children in the family, and a child with the ability to communicate needs to the hospital staff. Page Ref: 369

The adolescent is 6-hours postappendectomy and refuses pain medications. The nurse would like to walk the child in the hall but is concerned that the child has unrelieved pain. The nurse knows that unrelieved pain causes physiologic consequences such as (Select all that apply.) 1. Atelectasis 2. Pneumonia 3. Ileus 4. Lethargy 5. Hypoactive bowel sounds

Answer: 1, 2, 3 Page Ref: 315

The nurse is creating a teaching care plan for the toddler and family. Which nursing diagnoses are normally used at each healthcare visit for this age group? Select all that apply. 1. Knowledge deficit related to growth patterns 2. Risk for injury related to developmental skills 3. Risk for exposure to infectious diseases related to childcare environment 4. Knowledge deficit related to toys that encourage development 5. Risk for loneliness related to lack of siblings

Answer: 1, 2, 3, 4 Explanation: The toddler is assessed for height, weight, BMI, head circumference, growth and nutrition, verbal skills, gross and fine motor movement, appropriate toys for developmental age. Page Ref: 170

The preschool-age child has been back from surgery for removal of a Wilm's tumor for 6 hours, the nurse anticipates the preschooler will need pain medication very soon. The nurse is aware that the preschool-age child may not complain of pain because Select all that apply. 1. The preschooler cannot give a description of his pain. 2. The preschooler may assume the nurse knows he has pain. 3. The preschooler may be afraid it may hurt more to have the pain treated. 4. The preschooler believes he must be brave. 5. The preschooler uses sleeping to deal with pain.

Answer: 1, 2, 3, 4 Explanation: This is why the preschooler may not complain of pain. Children may not complain of pain for several reasons: young children cannot give a description of their pain because of a limited vocabulary or few pain experiences; some children believe they need to be brave and not worry their parents; preschoolers may assume the nurse knows they have pain, and some children are afraid that it will hurt more to have the pain treated. Page Ref: 313

A young school-age child is in the pediatric intensive-care unit (PICU) with a fractured femur and head trauma. The child was not wearing a helmet while riding his new bicycle on the highway and collided with a car. Which nursing diagnoses may be appropriate for this family? Select all that apply. 1. Guilt Related to Lack of Child Supervision and Safety Precautions 2. Family Coping: Compromised, Related to the Critical Injury of the Child 3. Parental Role Conflict Related to Child's Injuries and PICU Policies 4. Knowledge Deficit Related to Home Care of Fractured Femur 5. Anger Related to Feelings of Helplessness

Answer: 1, 2, 3, 5 Explanation: 1. All of these nursing diagnoses except Knowledge Deficit are possible in this situation. Although planning for discharge begins with admission, it is too early to begin teaching the parents about home care. The astute and experienced PICU nurse is prepared to recognize current problems and intervene appropriately. Page Ref: 266

The nurse is planning care for a preschool-age client who has cerebral palsy (CP). Which interventions are appropriate for this client? Select all that apply. 1. Providing heath supervision 2. Collaborating with physical therapy 3. Assisting with planning educational services 4. Prescribing medication for spasticity 5. Promoting growth and development

Answer: 1, 2, 3, 5 Explanation: Appropriate interventions for the nurse who is providing care to a client with a chronic condition include providing health supervision, collaborating with other specialties, assisting with planning educational services, and promoting growth and development. It is outside the scope of nursing practice to prescribe medication. The nurse could, however, administer prescribed medications if appropriate. Page Ref: 249

The nurse is assessing an adolescent and notes signs and symptoms of anorexia nervosa. Which signs and symptoms led the nurse to believe the adolescent has this condition? Select all that apply. 1. Extreme weight loss 2. Depression 3. Irregular menses 4. Sedentary lifestyle 5. Bradycardia

Answer: 1, 2, 3, 5 Page Ref: 301-303

The school nurse is teaching a class about safety. The nurse will teach the children that they should wear protective athletic gear when participating in selected activities. Which of these activities require protective athletic gear? Select all that apply. 1. Skateboarding 2. Playing football 3. Swimming 4. Playing lacrosse 5. Performing acrobatic tricks

Answer: 1, 2, 4 Explanation: Any sport that includes body contact requires a child to wear protective equipment. These include skateboarding, football, and lacrosse. Swimming and acrobatics do not have any requirements for protective equipment. Page Ref: 192

It is important that parents of adolescents with special needs transition care of the adolescent so they can learn to make good decisions on their own. Which items are considered transitional needs? Select all that apply. 1. Attending school 2. Discussing sexual matters 3. Letting most friends know of the medical condition 4. Socialization beyond the family 5. To write his or her own individualized healthcare plan

Answer: 1, 2, 4 Explanation: Transitional needs toward independence include attending school, discussion of sexual matters, and socialization beyond the family. The other areas are not transitional needs. Page Ref: 257-258

The nurse in the long-term care clinic is reviewing the charts of a group of children with chronic physical, psychological, functional, and social limitations. Which conditions are most likely to lead to chronic limitations? Select all that apply. 1. Near drowning 2. Congenital heart defect 3. Sinusitis 4. Fetal insult when the mother contracted rubella in the first trimester of pregnancy 5. Sepsis contracted as a neonate

Answer: 1, 2, 4, 5 Explanation: 1. All of these conditions or events except sinusitis can leave a child with a permanent chronic condition. Page Ref: 247-248

The nurse is asked to teach injury prevention measures to a classroom of 4-year-old preschoolers. Which teaching points are most appropriate at this age? Select all that apply. 1. Stop, drop and roll if clothes catch fire 2. Never go into the road alone. 3. Acceptable places for climbing 4. Safe meeting place outside the house in case of fire 5. Car seat safety

Answer: 1, 2, 4, 5 Explanation: Acceptable places to climb should be introduced in the toddler years when children are learning to walk, climb, and explore. It is not a topic for a preschool class. All the other topics are appropriate for this age. Page Ref: 175

The nurse manager is assisting the organization to open a healthcare center. What items must the manager include in pediatric inventory? Select all that apply. 1. Preprinted drug dosage chart 2. Oxygen face masks 3. Pediatric chairs and litters 4. Length-based resuscitation tape 5. Oral and NG airways and laryngoscope blades

Answer: 1, 2, 4, 5 Explanation: Essential equipment: child/neonate sized, proper weight dosage medications, smaller bags of IV fluids and special IV tubing, pediatric external defibrillator, pediatric oxygen masks/oral/NG airways and laryngoscope blades, length based resuscitation tapes and preprinted dosage chart to quickly identify equipment sizes and drug dosages by the length or weight of the child, essential emergency pediatric drugs and equipment. Page Ref: 210

The nurse is preparing to complete a health surveillance appointment with a school-age client and parents. Which observations would necessitate the need for further assessment by the nurse? Select all that apply. 1. Client who does not make eye contact 2. Client with visible bruises in various stages of healing 3. Client holding a video game talking with parent 4. Client playing a card game with sibling 5. Client who appears red in the face while walking to exam room

Answer: 1, 2, 5 Explanation: Nursing assessment begins with the first encounter with the client and the family. The nurse would want to further explore a client who does not make eye contact, who has bruises in various stages of healing, and a client who appears red in the face while walking to the exam room. All of these items may be clues to emotional issues, physical violence, and health related issues, such as hypertension. A client who is holding a video game and talking to the parent and a client who is playing a card game with a sibling are not observations that are abnormal for the school-age client. Page Ref: 184, 185

The emergency-room nurse receives a preschool-age child who was hit by a car. Which nursing interventions are a priority for this child? Select all that apply. 1. Performing a rapid head-to-toe assessment 2. Recording the parents' insurance information 3. Assessing airway, breathing, and circulation 4. Asking the parents about organ donation 5. Asking the parents if anyone witnessed the accident

Answer: 1, 3 Explanation: Assessing airway, breathing, and circulation and performing a rapid head-to-toe assessment are the priority nursing interventions. Asking the parents about organ donation is insensitive until the extent of the child's injuries is known. Recording insurance information is necessary but should never come before lifesaving assessment and intervention. Detailed information about the accident is helpful in determining the child's point of impact with the car and mechanism of injury, but this is not the initial priority. Page Ref: 262

While the nurse is conducting the history of a school-age child, the parents admit to owning firearms. Which safety measures are appropriate to include in the teaching plan for this family? Select all that apply. 1. Using a gun lock on all firearms in the house 2. Taking the child to a shooting range for lessons on how to use a gun properly 3. Storing the guns and ammunition in separate places 4. Keeping all the guns in a locked cabinet 5. Explaining the dangers of a gun to the child and telling her explicitly to never touch it

Answer: 1, 3, 4 Explanation: 1. Over 4000 youth from 10 to 19 years old die from firearm homicides annually, and approximately 1500 additional youth die from firearm suicide. Firearm homicide is the second leading cause of injury death for youth, and firearm suicide is the fifth leading cause of injury death for youth (CDC, 2011c). The safety measures of using a gun lock, keeping the gun and ammunition separate, and putting the guns in a locked cabinet will at least make the guns less accessible. Telling a child that a gun is "dangerous" and not to be touched will probably make it more fascinating. Even with knowledge of the proper use of a firearm, a 10-year-old child's judgment may not be mature enough to prevent misuse of it. Page Ref: 385

The nurse is admitting a school-age Vietnamese client who hit a parked car while riding a bike. The child has a fracture of the left radius and femur in addition to a fractured orbit. The child is stoic and denies pain. Which nursing actions are most appropriate in this situation? Select all that apply. 1. Use the FLACC scale to determine the child's pain level. 2. Tell the child to ring the call bell if the leg starts hurting. 3. Administer pain medication now and continue on a regular basis. 4. Ask the child's parents to notify the nurse if the child complains of pain. 5. Use the NIPS scale to determine the child's pain level.

Answer: 1, 3, 4 Explanation: Based on the type of injuries the child has, pain will be present. Analgesics should be given on a scheduled basis so that the pain does not get out of control. The FLACC scale is the most appropriate tool to use with an 8-year-old. The child's stoic expression is likely to be culturally related, and the child may not admit hurting. While asking the parents to call the nurse is not inappropriate, it is not the most appropriate initial action. The NIPS scale is appropriate for a newborn, not a school-age, client. Page Ref: 313-317

Which stressor is common in the hospitalized toddler with a chronic disorder? Select all that apply. 1. Fear of painful procedures 2. Self-concept 3. Interruption of normal routines 4. Unfamiliarity of caregivers 5. Isolation

Answer: 1, 3, 4 Page Ref: 252

The nurse is providing care to a preschool-age client who was admitted to the medical-surgical unit after an acute asthma attack. Which interventions foster a family-centered focus to client care? Select all that apply. 1. Discussing rooming in with the parents of the client 2. Allowing the client to "cry it out" after the parents leave for the evening 3. Providing comfort items from home, such as a blanket 4. Maintaining strict visitation for the family 5. Discussing what to expect during the hospital stay

Answer: 1, 3, 5 Explanation: 1. Family-centered care principles that are used in the hospital setting include rooming in, providing comfort items from home, and discussing what to expect. Allowing the child to "cry it out" and maintaining strict visitation for the family are not family-centered principles. Page Ref: 230-231

The parents of a toddler are concerned about their child's finicky eating habits. While counseling the parents, which statements by the nurse are the most appropriate? Select all that apply. 1. "The child is experiencing physiologic anorexia, which is normal for this age group." 2. "A general guideline for food quantity at a meal is one-quarter cup of each food per year of age." 3. "It is more appropriate to assess a toddler's nutritional demands over a 1-week period rather than a 24-hour one." 4. "Nutritious foods should be made available at all times of the day so that she is able to 'graze' whenever she is hungry." 5. "The toddler should drink 16 to 24 ounces of milk daily."

Answer: 1, 3, 5 Explanation: Physiologic anorexia is caused when the extremely high metabolic demands of infancy slow to keep pace with the slower growth of toddlerhood, and it is a very normal finding at this age. It is not unusual for toddlers to have food jags during which they only want one or two food items for that day. So it is more helpful to look at what their intake has been over a week instead of a day. Two to three cups of milk per day are sufficient for a toddler, and more than that can decrease the child's desire for other foods and lead to dietary deficiencies. The correct general guideline for food quantity is one tablespoon of each food per year of age. Food should only be offered at meal and snack times, and children should sit at the table while eating to encourage their socialization skills. Page Ref: 287

The school nurse plans, develops, manages, and evaluates healthcare services to all children while they are in the educational setting. With which healthcare providers will the nurse be collaborating? Select all that apply. 1. School physician 2. Teachers 3. Cafeteria staff 4. Primary physician 5. Bus driver

Answer: 1, 4 Page Ref: 211, 212

Match the behaviors with its stage of separation anxiety the child may exhibit. A. Protest B. Despair C. Denial 1. Withdrawal or compliant behavior 2. Appearance of being happy and content with everyone 3. Clinging to parents 4. Lack of protest when parents leave 5. Screaming and crying 6. Sadness

Answer: 1/B, 2/C, 3/A, 4/C, 5/A, 6/B 1. Despair 2. Denial 3. Protest 4. Denial 5. Protest 6. Despair Explanation: The stages of separation anxiety include: Protest-Screaming, crying, clinging to parents, and may resist attempts by other adults to comfort them. Despair-Sadness, quiet, appear to have "settled in," withdrawal or compliant behavior, and crying when parents return. Denial-Lack of protest when parents leave, appearance of being happy and content with everyone, show interest in surroundings, and close relationships not established. Page Ref: 227

A child is admitted with a diagnosis of early localized Lyme disease. Which clinical manifestations would the nurse expect to find on the initial assessment of this client? Select all that apply. 1. Erythema 5 to 15 cm in diameter 2. Hyperactivity 3. Cranial nerve palsies 4. Fever 5. Headache

Answer: 1, 4, 5 Explanation: Erythema, fever, and headache are signs/symptoms in the early localized stage of Lyme disease. Cranial nerve palsies are seen in the early disseminated stage of the disease. Malaise, rather than hyperactivity, is seen with this disease. Page Ref: 360

The family and school-age child are at the healthcare clinic for immunizations. The nurse takes the time to talk with the child and family about reducing the transmission of infection. What practices should the nurse suggest for the family? Select all that apply. 1. Do not share dishes, utensils, and cups. 2. Sanitize toys every week with Lysol. 3. Use alcohol-based hand sanitizer with the child after eating and toileting. 4. Cough or sneeze into cloth tissue 5. Dispose of diapers in a closed container.

Answer: 1, 5 Explanation: Teach families to reduce transmission of infection among family members with the following practices: use disposable tissues and dispose immediately after using, wash hands thoroughly with soap/water after all contact with diapers/tissues/mucous, sneeze/cough into elbow, wash hands with soap/water after eating and toileting, do not share dishes/utensils/cups, wash hands thoroughly before preparing food and again several times during the preparation process, use soapy warm water to wash dishes/cutting boards, wipe counters/surfaces that are used for diaper changes or that the child touches with disinfectant, make sure diaper changing area is well away from food prep areas, dispose of diapers in closed containers. This is a practice that the nurse should suggest for the family. Page Ref: 336

Match the child's concept of death with their behavioral response. A. Infant B. Toddler C. Preschool-age child D. School-age child E. Adolescent 1. Understands difference between temporary separation and death. 2. Senses emotions of caregivers and altered routines. 3. Capable of understanding death, recognizes all people and self will die. 4. No understanding of true concept of death. 5. Believes death is temporary and the person will return.

Answer: 1/D, 2/A, 3/E, 4/B, 5/C 1. School-age child 2. Infant 3. Adolescent 4. Toddler 5. Preschool-age child Explanation: School-age child—Understands difference between temporary separation and death. Infant—Senses emotions of caregivers, and altered routines. Adolescent—Capable of understanding death, recognizes all people and self will die. Toddler—No understanding of true concept of death. Preschool-age child— Believes death is temporary and the person will return. Page Ref: 278

Match the formalized plan for the child with a chronic condition with its description. A. Individualized family service plan (IFSP) B. Individualized education plan (IEP) C. Individualized health plan (IHP) D. Individualized transition plan (ITP) 1. Helps individuals receive vocational training and move successfully from the home into other community settings. 2. Developed for a child with cognitive, motor, social, and communication impairment who needs special education services. 3. Developed for the early intervention process for infants with special healthcare needs and their families. 4. Developed for the child with medical conditions that need to be managed within the school setting.

Answer: 1/D, 2/B, 3/A, 4/C 1. Individualized transition plan (ITP) 2. Individualized education plan (IEP) 3. Individualized family service plan (IFSP) 4. Individualized health plan (IHP) Explanation: Because some children need medications or other therapies during school hours, the parents and child, school nurse, teacher, and school administrators develop a plan to manage the child's condition during school hours. Page Ref: 255-256

The nurse has set up a group discussion for several families with chronically ill children. The nurse informs these parents that they may face which ethical issue? 1. Normalization 2. Withholding and refusal of treatment 3. Repeated hospital admissions 4. Lack of proper dietary needs

Answer: 2 Explanation: Withholding and refusal of treatment is an ethical issue involving the life and quality of life of the child. Normalization is a family process of adaptation as the family members cope with daily life with their child. Lack of dietary needs is not an ethical issue, nor is repeated hospital admissions. Page Ref: 253

A child is admitted to the neonatal intensive care unit (NICU). The parents are concerned because they cannot stay for long hours to visit. Which statement made by the nurse is most appropriate? 1. "One of you might take a leave of absence to be here more." 2. "Parents often feel this way; would you be interested in talking with others who have experienced having a child in the NICU?" 3. "Perhaps the grandparents can make the visits for you." 4. "Why can't you visit after work every day?"

Answer: 2 Explanation: "Parents often feel this way; would you be interested in talking with others who have experienced having a child in the NICU?" is therapeutic; it focuses on feelings and offers support to the parents. The other options do not focus on how the parents feel and attempt to solve the issue rather than allow for the parents to deal with their feelings and form solutions. Page Ref: 268

The parents of a critically injured child wish to stay in the room while the child is receiving emergency care. Which action by the nurse is the most appropriate? 1. Escort the parents to the waiting room and assure them that they can see their child soon. 2. Allow the parents to stay with the child. 3. Ask the physician if the parents can stay with the child. 4. Tell the parents that they do not need to stay with the child.

Answer: 2 Explanation: 1. Parents should be allowed to stay with their child if they wish to do so. This position is supported by the Emergency Nurses Association and is a key aspect of family-centered care. Page Ref: 227-228

An adolescent client has a stiff neck, a headache, a fever of 103 degrees Fahrenheit, and purpuric lesions noted on the legs. Although the adolescent's physical needs take priority at the present time, the nurse can expect which to be the most significant psychological stressor for this adolescent? 1. Separation from parents and home 2. Separation from friends and permanent changes in appearance 3. Fear of painful procedures and bodily mutilation 4. Fear of getting behind in schoolwork

Answer: 2 Explanation: Adolescents are developing their identity and rely most on their friends. They are concerned about their appearance and how they look compared to their peers. Separation from parents and home is the main psychological stressor for infants and toddlers. Preschool-age children fear pain and bodily mutilation. School-age children are developing a sense of industry and fear getting behind in schoolwork.

A child is admitted to the hospital unit with physical injuries. The nurse is taking the child's history. Which statement by the parent would arouse suspicion of abuse? 1. "I did not realize that my baby was able to roll over yet, and I was just gone a minute to check on dinner when the baby rolled off of the couch and onto our tile floor." 2. "The baby's 18-month-old brother was trying to pull the baby out of the crib and dropped the baby on the floor." 3. "I placed the baby in the infant swing. His 6-year-old brother was running through the house and tripped over the swing, causing it to fall." 4. "I was walking up the steps and slipped on the ice, falling while carrying my baby."

Answer: 2 Explanation: All of the statements made by the parent are plausible from a developmental perspective except the statement "The baby's 18-month-old brother was trying to pull the baby out of the crib and dropped the baby on the floor." Developmentally, it would be very difficult for an 18-month-old child to pull an infant out of a crib. Page Ref: 389-390

The nurse is providing care for several pediatric clients. Which client would require an Individualized Health Plan (IHP) prior to returning to school? 1. A school-age client who has recently developed a penicillin allergy 2. An adolescent client newly diagnosed with insulin-dependent diabetes mellitus 3. A school-age client who has been treated for head lice 4. An adolescent client who has missed two weeks of school due to mononucleosis

Answer: 2 Explanation: An IHP that ensures appropriate management of the child's healthcare needs must be developed for a child newly diagnosed with a chronic illness such as diabetes. A child who is allergic to penicillin will not receive this medication any longer and therefore should not encounter any problems related to it at school. A child who has been treated for head lice can return to school and does not need an IHP. While a child who has missed two weeks of school will need to make arrangements for makeup work, an IHP is not needed. Page Ref: 211

A nurse is talking to the mother of an exclusively breastfed African American 3-month-old infant who was born in late fall. Which supplement will the nurse recommend for this infant? 1. Iron 2. Vitamin D 3. Fluoride 4. Calcium

Answer: 2 Explanation: An infant's iron stores are usually adequate until about 4 to 6 months of age. The infant should be receiving sufficient amounts of calcium from breast milk, and fluoride supplementation, if needed, does not begin until the child is approximately 6 months old. This infant will have limited exposure to sunlight and thus vitamin D because of the infant's dark skin and decreased sun exposure in the fall and winter months. Page Ref: 284

A 2-month-old infant with bronchopulmonary dysplasia (BPD) is being prepared for discharge from the neonatal intensive-care unit (NICU). The infant will continue to receive oxygen via nasal cannula at home. Prior to discharge, the home-health nurse assesses the home. Which finding poses the greatest risk to this infant? 1. Small toys strewn on the floor 2. A woodstove used for heating 3. A sibling who has an ear infection 4. Paint peeling on the walls

Answer: 2 Explanation: Assessment of the home environment is essential prior to discharge of a medically fragile infant. The use of a woodstove poses great risk to the infant who already has fragile lungs. Oxygen and woodstove heat will produce a flammable reaction. Small toy pieces and paint peeling from the wall will pose a choking risk to the older infant who is crawling. Ear infections are not contagious. Page Ref: 214, 215

A hospitalized toddler-age client needs to have an IV restarted. The child begins to cry when carried into the treatment room by the mother. Which nursing diagnosis is most appropriate? 1. Ineffective Individual Coping Related to an Invasive Procedure 2. Anxiety Related to Anticipated Painful Procedure 3. Fear Related to the Unfamiliar Environment 4. Knowledge Deficit of the Procedure

Answer: 2 Explanation: At this age, the child is not old enough to understand the need for an IV infusion. The stem indicates that the child has been through this painful procedure before, and his reaction to entering the treatment room is based on anticipation of repeat discomfort. The child's behavior is appropriate for a child of this age. Page Ref: 319

During a clinic visit, the parents of a 15-month-old ask what disease and injury prevention topics would be appropriate to discuss at this age. Which response by the nurse is the most appropriate? 1. "It's never too early to teach a child to wear a helmet when riding a bicycle." 2. "Teaching simple handwashing is a good topic at this age." 3. "Tell the child over and over to stay away from water unless you are with him." 4. "Tell him firmly 'no' when he tries to cross the street."

Answer: 2 Explanation: Disease and injury prevention are ongoing topics at all ages. Simple handwashing is appropriate for a 15-month-old child. A 15-month-old is too young for bicycle riding, so this can be delayed. A 15-month-old is too young to understand water safety and crossing the street, and should never be left unattended in these situations. Page Ref: 175

A parent brings her school-age child to the clinic because the child has a temperature of 100.2°F. The child remains active without other symptoms. Which statement by the nurse to the parents is most appropriate? 1. "Take the child's temperature every 2 hours and call the clinic if it reaches 102°F or above." 2. "Unless the fever bothers the child, it is best to let the natural body defenses respond to the infection." 3. "Keep the child warm, because shivering often occurs with fever." 4. "Alternate acetaminophen and ibuprofen to help keep the fever down and keep the child comfortable."

Answer: 2 Explanation: Fever is the body's response to an infection, and is not a disease. Allowing the body's natural defenses (fever) to fight the infection is best. The fever is treated if the child is uncomfortable from effects of the fever, such as body aches, headache, and so on. Taking the child's temperature more than every 4 to 6 hours is unnecessary. The child should be dressed for comfort. Light clothing is recommended. Alternating acetaminophen and ibuprofen is not recommended. Page Ref: 349

As an advocate for the child undergoing bone-marrow aspiration, which intervention would the nurse suggest to decrease the pain experienced due to the procedure? 1. General anesthesia 2. Conscious sedation 3. Intravenous narcotics ten minutes before the procedure 4. Oral pain medication for discomfort after the procedure

Answer: 2 Explanation: For the child undergoing repeated procedures, it is important for the child to be sedated prior to and during the initial procedure. General anesthesia is not necessary for bone-marrow aspiration. Narcotics alone will not provide appropriate sedation to keep the child from remembering the procedure. While oral pain medication postprocedure is not inappropriate if discomfort exists, it is not the best answer. The child will have great anxiety and discomfort during the procedures and prior to future procedures. Page Ref: 330-331

Celiac disease presents many challenges for a family. What should the nurse emphasize when educating the parents of a newly diagnosed child? 1. Ice cream is a safe dessert on a gluten-free diet. 2. The child's weight and height should reach normal levels in about 1 year. 3. Processed foods are usually gluten-free. 4. Insurance pays only a small amount of the cost of celiac diets.

Answer: 2 Explanation: Ice cream and many processed foods contain gluten. Payment by insurance is dependent on the plan the family has. Once on a gluten-free diet, the child's height and weight will reach normal range in about 1 year. Page Ref: 298-299

An adolescent reports participating in an exercise program at school each Wednesday throughout the school year. Further history reveals that the adolescent does not participate in any other physical activities. Which outcome is most appropriate for this adolescent? 1. The adolescent is reporting information consistent with what 60 percent of adolescents report as participation in physical activities. 2. The adolescent is not meeting the recommendations of the Healthy People 2020 initiative. 3. The adolescent should be encouraged to continue this program of exercise, since something is better than nothing. 4. The adolescent should be encouraged to vigorously exercise for at least 5 minutes each day.

Answer: 2 Explanation: In this scenario, the adolescent is not receiving the recommended amount of exercise to support good health habits. Encouraging the adolescent to continue as is or to exercise vigorously for 5 minutes each day also is not consistent with current recommendations. Suggesting that "something is better than nothing" is not good practice. Page Ref: 185, 186

The nurse is providing care to a toddler-age child. Which assessment finding is indicative of abuse? 1. Parents indicating that they did not see the event occur 2. Inconsistency of stories between caregivers 3. Bruising noted on the knees and shins 4. Acting out behavior of the child

Answer: 2 Explanation: Inconsistency of stories is a red flag for abuse. All other answers are logical explanations for this age group. Page Ref: 389-390

The nurse is performing a well-child exam on a child who turned 4 years old 3 months ago. What can the nurse ask the child to do to assess appropriate milestones for this age? 1. Jump up and down 2. Throw a ball 3. Stack three or more blocks 4. Draw lines on paper

Answer: 2 Explanation: Jumping up and down, stacking three or more blocks, and drawing lines on paper are activities that represent milestones for young children. Throwing a ball and observing how it is thrown would assess a milestone for this age. By 4 to 5 years, a child begins to throw a ball overhand. Page Ref: 171, 172

The pediatric nurse is working with a parent who is suspected of Münchausen Syndrome by Proxy. Which action by the nurse is the priority? 1. Confront the parent with concerns of possible abuse. 2. Carefully document parent-child interactions. 3. Try to keep the parent separated from the child as much as possible. 4. Explain to the child that the parent is causing the illness and that the health team will prevent the child from being harmed.

Answer: 2 Explanation: Münchausen Syndrome by Proxy is very difficult to prove, and evidence provided by the careful documentation of the nursing staff can be very influential. Care must be taken not to make the parent suspicious and to keep the child in the hospital until enough evidence is collected. Confronting the parent or separating the parent from the child may alienate the parent and cause him or her to leave with the child. Talking to the child about the health-care team's suspicions may be confusing and frightening for the child. Page Ref: 394-395

While teaching a health promotion class to a group of parents of children in a Head Start class, which information should the nurse include to help decrease the risk of dental caries? 1. Delay introducing cow's milk until at least 1 year of age. 2. Offer drinking cups only at meal and snack times. 3. Encourage use of homemade baby food without preservatives. 4. Offer juices diluted 50 percent with water.

Answer: 2 Explanation: Offering drinking cups only at meal and snack times encourages drinking when thirsty rather than carrying a cup around. This reduces the risk of dental caries. Delaying the introduction of cow's milk, making homemade baby food, or diluting juice does not decrease dental caries. Page Ref: 284-285

A mother brings her 4-month-old infant in for a routine checkup and vaccinations. The mother reports that the infant was exposed to a brother who has the flu. Which action by the nurse is most appropriate based on these assessment findings? 1. Withhold the vaccinations. 2. Give the vaccinations as scheduled. 3. Withhold the DTaP vaccination but give the others as scheduled. 4. Give the infant the flu vaccination but withhold the others.

Answer: 2 Explanation: Recent exposure to an infectious disease is not a reason to defer a vaccine. There is no reason to withhold any of the vaccinations due at this time. The flu vaccination would not routinely be given to a 4-month-old. Page Ref: 349

The child was just transferred to the postanesthesia unit (PACU) and report given. The nurse has performed baseline vital signs, the child is stable and pain is under control. What should the nurse do next? 1. Document 2. Allow the parents to visit the child 3. Discharge the child 4. Look for signs of infection 5. Offer clear liquids

Answer: 2 Page Ref: 224

A parent asks the nurse if there is anything that can be done to reduce the pain that his 3-year-old experiences each morning when blood is drawn for lab studies. Which intervention would the nurse implement- based on the parent's concern? 1. Intravenous sedation 15 minutes prior to the procedure 2. EMLA cream (lidocaine 2.5% and prilocaine 2.5%) applied to skin at least one hour prior to the procedure 3. Use of guided imagery during the procedure 4. Use of muscle-relaxation techniques

Answer: 2 Explanation: Sedation is not generally used with quick minor procedures such as venipuncture. A 3-year-old is too young to participate in techniques such as muscle relaxation and guided imagery. EMLA cream is shown to be effective in providing topical anesthesia if applied at least one hour prior to the procedure. Page Ref: 327-328

The nurse is preparing to perform a heel stick on a neonate. Which complementary therapy is appropriate for the nurse to use decrease pain during this quick but painful procedure? 1. Swaddling 2. Sucrose pacifier 3. Massage 4. Holding the infant

Answer: 2 Explanation: Sucrose provides short-term natural pain relief and is most appropriate for use in neonates to decrease pain associated with a quick procedure. The other measures are more appropriate following the procedure or as an adjunct to pain medication for ongoing pain or distress. Page Ref: 323

A family actively participates in school functions. One of the children is paraplegic and requires a wheelchair for mobility. Which process does the nurse determine the family is working on based on these assessment findings? 1. Stagnation 2. Normalization 3. Isolation 4. Interaction

Answer: 2 Explanation: The family is normalizing life with the children through activities. The family is not staying at home because one member cannot walk; rather, the family is moving on to full participation in life. The family is interacting with others through the process of normalization. Page Ref: 255

The telephone triage nurse receives a call from a parent who states that her 18-month-old is making a crowing sound when he breathes and is hard to wake up. Which action by the nurse is the most appropriate? 1. Obtain the history of the illness from the parent. 2. Advise the parent to hang up and call 9-1-1. 3. Make an appointment for the child to see the healthcare provider. 4. Reassure the parent and provide instructions on home care for the child.

Answer: 2 Explanation: The nurse should immediately recognize the symptoms of severe upper respiratory distress and advise the parent to call 9-1-1. Crowing is heard when there is severe narrowing of the airway. The other actions would be appropriate in nonemergency situations. Page Ref: 209

Which intervention is considered supportive care for a family whose infant has died from sudden infant death syndrome (SIDS)? 1. Interviewing parents to determine the cause of the SIDS incident 2. Allowing parents to hold, touch, and rock the infant 3. Sheltering parents from the grief by not giving them any personal items of the infant, such as footprints 4. Advising parents that an autopsy is not necessary

Answer: 2 Explanation: The parents should be allowed to hold, touch, and rock the infant, giving them a chance to say good-bye to their baby. The other options are nontherapeutic. The death of an infant without a known medical condition is an indication for an autopsy. Page Ref: 277

A 9-year-old child who has been followed in the same pediatric home since birth is at the healthcare center for a well-child visit. A nurse who measures the height and weight of the child documents 35th percentile for height and 90th percentile for weight. How should the nurse interpret these data? 1. The child is beginning a growth spurt. 2. The child is obese and needs dietary counseling. 3. The parents are most likely below the 50th percentile for height and weight. 4. As soon as the child begins the adolescent growth spurt, the height and weight measurements will normalize.

Answer: 2 Explanation: These data show that the child is disproportionate in height and weight. This child's weight is very high in comparison to height. The child would appear obese. Dietary history and counseling are the first steps. This child may also need an endocrine evaluation. This is not a growth spurt since height is what is referred to as a growth spurt. No assumptions about the parents can be made from the data presented. The statement about the adolescent growth spurt is incorrect for a child of this age. Page Ref: 185, 186

A child is on a ventilator in the pediatric intensive care unit (PICU). Which nursing intervention would best meet the psychosocial needs of this child? 1. Allow the parents to remain at the bedside. 2. Touch and talk to the child often. 3. Provide the child with a blanket from home. 4. Provide consistent caregivers.

Answer: 2 Explanation: Touch and verbal exchanges will aid in psychosocial support. The other responses provide a sense of security. Page Ref: 267-269

The charge nurse on a hospital unit is developing plans of care related to separation anxiety. The charge nurse recognizes that which hospitalized child at highest risk to experience separation anxiety when parents cannot stay? 1. 6-month-old 2. 18-month-old 3. 3-year-old 4. 4-year-old

Answer: 2 Explanation: While all of these children can experience separation anxiety, the young toddler is at highest risk. Toddlers are the group most at risk for a stressful experience when hospitalized. Separation from parents increases this risk greatly. Page Ref: 224

While working at a weekend "free clinic," the nurse is assessing a toddler when the mother of the child confides that it has been very difficult providing for her family of four children on her limited budget. She is not sure that she has enough money to buy food for the rest of the month and the antibiotic that is needed for the child's ear infection. Which intervention would be the most beneficial for this family? 1. Giving the mother enough free samples of the antibiotic for the recommended course of treatment 2. Putting the mother in contact with a local agency that provides food on a regular basis to needy families and helps them access other resources in the community 3. Talking with the mother about the factors that increase a child's risk of acquiring an ear infection 4. Talking with the mother about the importance of a balanced diet in the growth and development of children and providing her with a list of inexpensive, nutritious foods

Answer: 2 Explanation: While most of these are good interventions, putting the mother in contact with a local agency is most likely to meet the family's basic need for food and possibly connect the mother to a resource that could supply her with the antibiotic for her child. Also, many uncomplicated ear infections can resolve without antibiotic treatment. Giving antibiotics will only benefit the child, and the question asks what will benefit the child and family. Page Ref: 371

A recently divorced mother who must return to work is concerned about the effects of placing her child in day care full time. In counseling the mother, which factor does the nurse share as the most influential in determining whether or not day care has a positive or negative effect on the child? 1. The ratio of day-care workers to children 2. The closeness of the parent-child relationship 3. The amount of time that the children spend playing outside 4. The cleanliness of the facility

Answer: 2 Explanation: While the ratio of child-care workers to the children, the cleanliness of the facility, and how much time the children are able to spend playing outdoors all can contribute to whether or not child care is a positive or negative experience, the closeness of the parent-child relationship is more likely to impact how resilient the child is, and this has a greater impact on the effects of the child-care experience. Page Ref: 372

The community-health nurse is planning an education session for recently hired teachers at a child-care center. Which item is priority for the community-health nurse to include in the educational session? 1. The schedule for immunizations 2. Principles of infection control 3. How to interpret healthcare records 4. How to take a temperature

Answer: 2 Explanation: 1. While all of the information is nice to know, it is most essential that teachers know principles of infection control to decrease the spread of germs that can cause disease in young children. Page Ref: 209

The nurse is providing nutritional guidance to the parents of a toddler. Which comment by the parent would prompt the nurse to provide additional education? 1. "I should not give my child raw oysters." 2. "It is safe to leave my meat red in the center as long as there are no juices running." 3. "We always wash our hands well before any food preparation." 4. "We use separate utensils for preparing raw meat and preparing fruits, vegetables, and other foods."

Answer: 2 Explanation: Meats should be cooked thoroughly before eating. Meat that is red in the center, with or without running juices, is insufficiently cooked and increases the risk of food-borne illness. Washing hands and using separate utensils help to prevent infection with food-borne pathogens. Raw oysters should be avoided. Page Ref: 296-297

A mother reports that her adolescent is always late. The mother states, "She was born late and has been late every day of her life." Which response should the nurse make to this mother? 1. "You need to establish specific time frames for your adolescent and be certain she adheres to them." 2. "You should not expect your adolescent to be an 'on-time' individual unless you set specific alarms and then reinforce the value of being 'on-time.'" 3. "You should not expect your adolescent to be on time. Teenagers are always late." 4. "You have a major problem. There must be a lot of screaming in your home."

Answer: 2 Explanation: The best response is to help the mother find a way to help the teen deal with the problem of lateness. The other responses will either create parent-child conflict or make assumptions about household communication. Page Ref: 190, 191

A school-age client who recently moved to a new school in a different town presents to an ambulatory care center and describes the following: "I have no friends in my new school and I no longer want to play soccer. I know I will be lonely there, too." Which of these takes priority when speaking with the school-age client? 1. Helping the school-age client realize the value of soccer 2. Promoting healthy mental-health outcomes 3. Acknowledging the fact that it takes several months to make new friends at a new school 4. Stressing the importance of remaining in a close parent-child relationship during these stressful times

Answer: 2 Explanation: The school-age client is obviously lonely with the move to the new school. The nurse should focus on appropriate coping skills, which will enhance good mental-health outcomes for the child. It would not be appropriate to discuss the importance of soccer at this time, since the school-age client must deal with the loss of friends and developing new friendships first. The parent-child relationship should not be used as a substitute for the development of new peer relationships. Page Ref: 185, 186

In working with parents of children with chronic diseases, the nurse is concerned with helping the parents to protect themselves from compassion fatigue. Which activities are appropriate for the nurse to encourage? Select all that apply. 1. Sleeping more than 9 hours per 24-hour period 2. Exercising 3. Fostering social relationships 4. Developing a hobby 5. Moving away

Answer: 2, 3, 4 Explanation: 1. Exercising, fostering social relationships, and developing a hobby all contribute to physical, spiritual, social, and mental rest and restoration. Sleeping more than the body requires and moving away are avoidance behaviors that do not address exhaustion from overwhelming caregiving responsibilities. Page Ref: 258

The nurse is assessing the toddler for early childhood caries. The nurse will teach the family which factors contribute to this condition? Select all that apply. 1. Inadequate activity 2. Inadequate dental care 3. Inadequate diet 4. Inadequate brushing 5. Inadequate pacifiers

Answer: 2, 3, 4 Explanation: Early childhood caries is promoted by inadequate preventive care, which can include diet, brushing, feeding habits, and lack of dental care. ECC is serious because young children with the condition are more likely to have continuing dental problems that can influence speech, cause pain, and delay development. Page Ref: 172

Which nursing interventions would be best for the nursing diagnosis of Powerlessness Related to Relinquishing Control to the Healthcare Team? Select all that apply. 1. Provide a primary nursing care model. 2. Prepare the child in advance for procedures. 3. Provide optimal pain relief. 4. Explain procedures in developmentally appropriate terms. 5. Incorporate home rituals when possible.

Answer: 2, 4, 5 Explanation: Preparation in advance—and in terms that are developmentally appropriate—and incorporating home rituals provide some degree of control, and might reduce the feeling of powerlessness. Providing a primary nursing care model will help decrease anxiety, and providing pain relief will decrease pain. Page Ref: 273-274

What are some common health problems associated with the poor and/or homeless child? Select all that apply. 1. Asthma 2. Sexually transmitted infections 3. Good dentition 4. Mental illness 5. Tuberculosis

Answer: 2, 4, 5 Page Ref: 371

A nurse working in a pediatric clinic is responsible for monitoring and maintaining the vaccinations on site. Which actions are appropriate for this nurse to implement? Select all that apply. 1. Fluctuate refrigerator and freezer temperatures each day. 2. Store vaccines in the center of the unit. 3. Check and record the temperature of the unit twice each day. 4. Have a plan for power outages. 5. Place bottles of water in each unit to help keep temperatures consistent.

Answer: 2, 3, 4, 5 Explanation: Appropriate interventions for the nurse to implement in order to maintain the potency of vaccines include storing the vaccines in the center of the unit, checking and recording the temperature of the storage unit twice a day, having a plan for power outages, and placing bottles of water in each unit to help keep temperatures consistent. The temperature of the refrigerator and freezer should be consistent and not fluctuate. Page Ref: 346

The nurse is teaching a prenatal class about infant care. Under which circumstances should the nurse emphasize that parents should call their healthcare provider immediately? Select all that apply. 1. Child 4 months old, received a DTaP immunization yesterday, and has a temperature of 38.0°C (100.4°F) 2. Child under 3 months old and has a temperature over 40.1°C (104.2°F) 3. Child difficult to awaken and has a pulsing fontanel 4. Child has purple spots on the skin and is lethargic. 5. Child has a stiff neck and has been irritable for three days.

Answer: 2, 3, 4, 5 Explanation: Infants under 3 months of age have limited ability to develop antibodies to fight infection, and a fever as high as 40.1°C indicates a serious infection. Difficulty to awaken and a pulsing fontanel, purple spots on the skin and lethargy, a stiff neck and irritability for 3 days in infants and children of any age may indicate meningitis. A mild fever of 38.0°C (100.4°F) in the 4-month-old who received a DTaP immunization yesterday is incorrect because the mild fever is expected as the body develops antibodies in response to antigens in the immunization. Page Ref: 362

Which health promotion activities can the nurse recommend to the parents of a preschool-age child in order to enhance the child's self-concept? Select all that apply. 1. Encourage a play date with a school-age child. 2. Praise the child for staying dry at night. 3. Tell the child there will be a punishment for bathroom accidents. 4. Set aside time for the child each day. 5. Discuss appropriate activities to engage in with the daycare provider.

Answer: 2, 4, 5 Explanation: Health promotion activities focus on development of a healthy self-concept in the toddler and young child by helping parents to set up successful play experiences, to praise the child for successes, to use effective limit-setting techniques, and to realize and appreciate the child's unique characteristics. Health maintenance seeks to avoid the poor self-image that can occur with constant criticism or expectations not in alignment with the toddler's or preschooler's developmental capabilities. Page Ref: 173, 174

In counseling an adolescent female about safe sex practices, which question is the most appropriate for the nurse to ask? 1. "Do you and your boyfriend use a condom every time you have sex?" 2. "Do you have a boyfriend, and if so, are you sexually active?" 3. "Do you have one or more sexual partners?" 4. "Have you and your boyfriend ever had unprotected sex?"

Answer: 3 Explanation: 1. "Do you have one or more sexual partners?" provides the adolescent an opportunity to discuss sexual practices in a homosexual or alternative relationship if this applies to her. All of the other questions assume that the adolescent is involved in heterosexual relationships. Page Ref: 370

The nurse needs to administer a medication to a preschool-age child. The medication is only available in tablet form. Which action by the nurse is the most appropriate? 1. Place the tablet on the child's tongue and give the child a drink of water. 2. Break the tablet in small pieces and ask the child to swallow the pieces one by one. 3. Crush the tablet and mix it in a teaspoon of applesauce. 4. Crush the table and mix it in a cup of juice.

Answer: 3 Explanation: 1. A 4-year-old is not mature enough to swallow a pill or pieces of a pill. The medication should be crushed and mixed with a very small amount of food, not juice. Page Ref: 234

An adolescent female presents at a nurse practitioner's office and requests a signature for working papers. The nurse reviews her chart and notes that the last physical examination was two years ago. In addition to providing the signature for the working papers, what else should the nurse use this visit? 1. An opportunity to discuss birth-control measures 2. A time to discuss exercise and sports participation 3. A health-supervision opportunity 4. A chance to discuss the importance of pursuing post-secondary education

Answer: 3 Explanation: 1. All visits should be used as health-promotion and health-supervision visits. While discussing birth control, exercise, and future plans is important, these can be included in the overall health-supervision protocols. Page Ref: 183

A school-age client is being discharged from the outpatient surgical center. Which statement by the parent would indicate the need for further teaching? 1. "I can expect my child to have some pain for the next few days." 2. "I will plan to give my child pain medicine around the clock for the next day or so." 3. "Since my child just had surgery today, I can expect the pain level to be higher tomorrow." 4. "I will call the office tomorrow if the pain medicine is not relieving the pain."

Answer: 3 Explanation: 1. Increasing pain can be a sign of complication and should be reported to the physician; therefore, if the parent expects the pain to be higher the next day, the nurse should clarify expectations for pain control. The child is expected to have some pain for a few days after surgery and should receive pain medication on a scheduled basis. If prescribed medication is not relieving the pain to a satisfactory level, the physician should be notified. Page Ref: 328-329

The school nurse is preparing a plan of care specific to several children in the school who have asthma. What is the initial action on the plan of care? 1. Call 911 to request emergency medical assistance. 2. Call the child's parents to come and pick up the child. 3. Have the child use his or her metered-dose inhaler. 4. Have the child lie down to see if the symptoms subside.

Answer: 3 Explanation: A child with a history of asthma may have episodes of wheezing that can be controlled by prompt use of the child's rescue inhaler. An inhaler should be readily available in the school setting for a child previously diagnosed with asthma. This should be tried first. Emergency personnel should be notified if the inhaler does not provide relief and the child is in respiratory distress. Parents may be notified if the child does not feel well, but this is not the initial action. Having the child lie down will likely worsen his condition. Page Ref: 211, 212

Which client in the pediatric intensive care unit (PICU) would most benefit from palliative care? 1. A child with end-stage leukemia 2. A child with a broken arm after a motor vehicle accident 3. A child with burn injuries to the legs 4. A child with recurrent asthma

Answer: 3 Explanation: A child with burn injuries to the legs will benefit most from palliative care to help control pain, anxiety, sleep disturbances, and so on. The child with end-stage leukemia will benefit from hospice care. The child with a broken arm or recurrent asthma will not need palliative care. Page Ref: 263-264

The nurse is conducting a health promotion class for adolescents. In counseling an adolescent about lifestyle choices, what should the adolescent eliminate in order to decrease the risk of the most preventable cause of adult death? 1. Alcohol use 2. Obesity 3. Tobacco use 4. Cocaine use

Answer: 3 Explanation: Although all of these factors are preventable causes of mortality in the United States, tobacco use accounts for 438,000 deaths annually and is the most preventable cause of adult death. Page Ref: 375-376

A toddler client with a fever is prescribed amoxicillin clavulanate 250 mg/5 cc three times daily by mouth × 10 days for otitis media. Which teaching point will guard against antibiotic resistance to the disease process? 1. Administer a loading dose for the first dose. 2. Measure the prescribed dose in a household teaspoon. 3. Give the antibiotic for the full 10 days. 4. Stop the antibiotic if the child is afebrile.

Answer: 3 Explanation: Antibiotics must be administered for the full number of days ordered to prevent mutation of resistant strains of bacteria. A loading dose was not ordered. A household teaspoon may contain less than 5 cc, and the full dose must be given. Stopping the antibiotic before the prescribed time will permit remaining bacteria to reproduce, and the otitis media will return, possibly with antibiotic-resistant organisms. The absence of a fever is not an indication that all bacteria are killed or not reproducing. Page Ref: 362-364

The nurse is working in an adolescent medical clinic. What can the nurse anticipate when comparing adolescents in the clinic with chronic conditions to their peers? 1. A high level self-esteem 2. A concern for their parents 3. An altered body image 4. A decreased concern about their appearance

Answer: 3 Explanation: As adolescents develop a sense of identity, they are focused on themselves and the present. They have a heightened concern about their appearance but may have inaccurate assessments of their body image and low self-esteem when comparing their bodies with those of their peers. Page Ref: 252

The nurse is instructing a parent of a newborn on the foods that are to be started based on age. The nurse instructs the parent that the first food given to a newborn is rice cereal. What statement by the parent suggests appropriate understanding of the next food that can be introduced? 1. "Chicken can be given next." 2. "Eggs can be given next." 3. "Fruits should be given next." 4. "Whole milk should be started."

Answer: 3 Explanation: Chicken is not given until 8 to 10 months, eggs are not given until 12 months, whole milk is given at 12 months. Fruits are given after rice cereal. Page Ref: 286

A school-age client, recently diagnosed with asthma, also has a peanut allergy. The nurse instructs the family to not only avoid peanuts but also to carefully check food label ingredients for peanut products and to make sure dishes and utensils are adequately washed prior to food preparation. The mother asks why this is specific for her child. Based on the client's history, the nurse knows that this client is at an increased risk for which complication? 1. Urticaria 2. Diarrhea 3. Anaphylaxis 4. Headache

Answer: 3 Explanation: Children with food allergies may experience all of the above reactions to a particular food, but the child who also has asthma is most at risk for death secondary to anaphylaxis caused by a food allergy. Page Ref: 305

When examining a toddler-age child during a well-child physical, which assessment is the priority? 1. Visual acuity 2. Helmet use 3. Risk of lead exposure 4. Whether household drinking water contains fluorine

Answer: 3 Explanation: Elevated lead levels are neurotoxic to young children and, if untreated, can cause irreparable neurological damage. Visual acuity may be difficult to accurately assess at this age secondary to the child's compliance and ability to understand the directions for the screening test. While teaching helmet use at an early age is important, it is unlikely that this child is riding a bicycle yet, and although early exposure to fluorine is important for good dental health, lack of fluorinated drinking water will not be as harmful to the child as toxic lead levels. Page Ref: 400-401

At the conclusion of teaching parents about cerebral palsy, the nurse asks, "What is your hope for your toddler with cerebral palsy?" Which reply from a parent best indicates an understanding of a realistic achievement for the child? 1. "I hope my child qualifies for the Winter Olympics like I did." 2. "I hope my child just enjoys life." 3. "I hope my child will attend our neighborhood school." 4. "I hope my child is liked and accepted by other children."

Answer: 3 Explanation: Expecting a child with cerebral palsy to do well in the local school is a realistic hope that the child can possibly achieve. A child with cerebral palsy does not have the gross motor skills to qualify for the Olympics; thus, this is unrealistic. A hope for the child to enjoy life is realistic, but is not an achievement for the child. A hope that the child is liked and accepted by other children is realistic, but this hope is also dependent on other children. Page Ref: 253

A nurse obtains a nutritional health history from a 10-year-old child. Which of these food selections, if consumed on a regular basis, should lead the nurse to become concerned about the need for improving oral hygiene? 1. Peanuts and crackers 2. Sorbet and yogurt 3. Gummy bears and licorice 4. Fluoridated water

Answer: 3 Explanation: Food items that stick to the teeth lead to dental caries. Items such as gummy bears and licorice all stick to the teeth and lead to dental caries. Foods such as peanut butter, crackers, sorbet, and yogurt do not stick to the teeth and are not considered foods that increase dental caries. Fluoridated water has been shown to decrease the incidence of dental caries. Page Ref: 185

The nurse is caring for a client in the pediatric intensive-care unit (PICU). The parents have expressed anger over the nursing care their child is receiving. Which nursing intervention is most appropriate based on the situation? 1. Ask the physician to talk with the family. 2. Explain to the parents that their anger is affecting their child so they will not be allowed to visit the child until they calm down. 3. Acknowledge the parents' concerns and collaborate with them regarding the care of their child. 4. Call the chaplain to sit with the family.

Answer: 3 Explanation: Hospitalization of the child in a pediatric intensive-care unit is a great stressor for parents. If the parents feel that they are not informed or involved in the care of their child, they may become angry and upset. Calling the physician or chaplain may be appropriate at some point, but the nurse must assume the role of supporter in this situation to promote a sense of trust. Telling the parents that they cannot visit their child will only increase their anger. Page Ref: 227-229

Concerned parents call the school nurse because of changes in their 15-year-old adolescent's behavior. Which behavior would the nurse indicate as indicative of adolescent substance abuse? 1. Buying baggy, oversized clothing at thrift shops and dying her hair black 2. Becoming very involved with friends and in activities related to the basketball team that she is on; never seeming to be home; and, when she is home, preferring to be in her room with the door shut 3. Receiving numerous detentions lately from teachers for sleeping in class 4. Becoming very moody, dramatically crying and weeping one minute and then being cheerful and excited the next

Answer: 3 Explanation: Mood swings, experimenting with clothes and hair, periodically distancing themselves from their parents, and preferring involvement with their peers are all normal adolescent behaviors. Even though most teens do prefer staying up late, they are not usually so tired that they would fall asleep during the day, especially while engaged in classroom activities. This behavior is abnormal and may indicate involvement with substance abuse or an underlying pathology. Page Ref: 378

A nurse is providing information to a group of new mothers. Which statement best explains why newborns and young infants are more susceptible to infection? 1. "They have high levels of maternal antibodies to diseases to which the mother has been exposed." 2. "They have passive transplacental immunity from maternal immunoglobulin G." 3. "They have immune systems that are not fully mature at birth." 4. "They have been exposed to microorganisms during the birth process."

Answer: 3 Explanation: Newborns have a limited storage pool of neutrophils and plasma proteins to defend against infection. Newborns' and young infants' high levels of maternal antibodies, passive transplacental immunity, and exposure to microorganisms during the birth process are all true but are incorrect answers because they do not explain the susceptibility of newborns and young infants to infection. Page Ref: 334-335

A school-age child with congenital heart block codes in the emergency department (ED). The parents witness this and stare at the resuscitation scene unfolding before them. Which nursing intervention is most appropriate in this situation? 1. Ask the parents to leave until the child has stabilized. 2. Ask the parents to call the family to come into watch the resuscitation. 3. Ask the parents to sit near the child's face and hold her hand. 4. Ask the parents to stand at the foot of the cart to watch.

Answer: 3 Explanation: Parents should be helped to support their child through emergency procedures, if they are able. Parents should never be asked to take part in emergency efforts unless absolutely necessary. Merely watching the resuscitation serves no purpose for the child. If the parents interfere with resuscitation efforts or they are unable to tolerate the situation, they can be asked to leave later. Page Ref: 268

A preschool-age client is hospitalized following surgery for a ruptured appendix. During assessment of the child, the nurse notes that the child is sleeping. Vital signs are as follows: temperature 97.8 degrees F axillary, pulse 90, respirations 12, and blood pressure 100/60. Which conclusion by the nurse is appropriate based on the assessment findings? 1. The client is comfortable and the pain is controlled. 2. The client is in shock secondary to blood loss during surgery. 3. The client is experiencing respiratory depression secondary to opioid administration for postoperative pain. 4. The client is sleeping to avoid pain associated with surgery.

Answer: 3 Explanation: Respiratory depression secondary to opioid use is most likely to occur when the child is sleeping. A respiratory rate of 12 is well below normal for a preschool-age client. The other vital signs are within normal limits for a sleeping preschool-age client. Page Ref: 321

Some nursing students are discussing job options. One of the student states that a position as a school nurse sounds interesting. What is an important role of the school nurse? 1. Screening for congenital heart disease 2. Prescribing antibiotics for streptococcal pharyngitis 3. Developing a plan for emergency care of injured children 4. Diagnosing an ear infection

Answer: 3 Explanation: Screening of students for certain conditions; educating students, teachers, and staff; and developing emergency plans are all roles of the school nurse. Diagnosing acute illness and prescribing medication for a new illness are beyond the scope of practice for the school nurse unless the nurse is licensed as an advance-practice nurse. Page Ref: 211, 212

The nurse is evaluating the car seat of a 3-year-old who weighs 42 pounds. Which recommendation should the nurse make about the car seat to the parents? 1. Convertible, rear-facing seat 2. Belt-positioning booster seat 3. A car seat with a harness approved for higher weights and heights 4. A regular seat with lap and shoulder strap

Answer: 3 Explanation: The American Academy of Pediatrics and the National Highway Safety Administration recommend booster seats for children over 40 pounds and 4 years of age. A 3-year-old should be in a regular car seat with approved harness for higher-weight/height children so that the child is protected from injury. Rear-facing seats and regular seat with lap and shoulder strap are not appropriate for a 3-year-old. Page Ref: 175

The hospital has just provided its nurses with information about biologic threats and terrorism. After completing the course, a group of nurses is discussing its responsibility in relation to bioterrorism. Which statement by the nurse indicates a correct understanding of the concepts presented? 1. "It is important to separate clients according to age and illness to prevent the spread of disease." 2. "It is important to dispose blood-contaminated needles in the lead-lined container." 3. "I will notify the Centers for Disease Control (CDC) if a large number of persons with the same life-threatening infection present to the emergency room." 4. "I will initiate isolation precautions for a hospitalized client with methicillin-resistant staphylococcus aureus (MRSA)."

Answer: 3 Explanation: The CDC must be contacted to investigate the source of serious infections and to determine if a bioterrorist threat exists. Separating clients according to age and illness to prevent the spread of disease will do nothing to stop terrorism. Proper disposal of blood-contaminated needles in the sharps container and initiating isolation precautions for a hospitalized client with methicillin-resistant staphylococcus aureus (MRSA) are appropriate nursing actions but do not relate to bioterrorism. Page Ref: 365

During a well-child physical, an adolescent female has a normal history and physical except for an excessive amount of tooth enamel erosion, a greater-than-normal number of filled cavities, and calluses on the back of her hand. Her body mass index is in the 50th to 75th percentile for her age. Which disorder is the nurse concerned about based on the assessment findings? 1. Anorexia nervosa 2. Kwashiorkor 3. Bulimia nervosa 4. Marasmus.

Answer: 3 Explanation: The erosion of tooth enamel, dental caries, and calluses on the back of her hand all most likely are due to frequent vomiting of gastric acids, which is common with bulimia nervosa as part of a binge-purge cycle. Anorexia nervosa is an eating disorder where adolescents literally starve themselves to prevent weight gain; they also exercise excessively and use laxatives and diuretics to lose weight. Anorexia usually manifests as extreme weight loss and an obsession with food. Kwashiorkor is a protein deficiency, usually from malnutrition, that manifests as generalized edema. Marasmus is a lack of energy-producing calories that can be seen in anorexia, and this causes emaciation, decreased energy levels, and retarded development. Page Ref: 303-304

An obese adolescent who adamantly denies sexual activity has a positive pregnancy test, which was performed in the adolescent clinic. Which statement by the nurse is the most appropriate in this situation? 1. "Tell me how you feel about your body image." 2. "When was your last menstrual period (LMP)?" 3. "Let's discuss some activities that you have done within the past few months that could possibly lead to pregnancy." 4. "Were you involved in a date rape and are you hesitant to speak about it?"

Answer: 3 Explanation: The nurse must help the adolescent realize that previous behaviors have led to a positive pregnancy test. The only response by the nurse that will accomplish this goal is for the nurse to ask a direct question in which the nurse and client search for an answer. Page Ref: 189

The nurse is discussing ways to treat fever in the home environment to a group of parents in the community. Which statement is appropriate for the nurse to include in the presentation? 1. "Ibuprofen is the only effective means to reduce fever." 2. "If the child requires more than one dose of acetaminophen antibiotics are needed." 3. "Purchase a new bottle of acetaminophen for your newborn because it will have recommended medication concentration." 4. "It is not necessary to follow the recommendations on the bottle of ibuprofen as this will not prevent an overdose for your child."

Answer: 3 Explanation: The recommendation to purchase a new bottle of acetaminophen due to recommended medication concentrations is an appropriate statement for the nurse to include in the teaching session. The other statements are inaccurate or inappropriate for the nurse to include in the teaching session. Page Ref: 349

A social service coordinator is consulted to arrange for a phototherapy blanket at discharge for an infant/family with multiple social difficulties. Which social difficulty is more than likely to have the greatest influence on discharge? 1. Cultural practices and rituals 2. Financial difficulties 3. The family is homeless 4. The family does not have a healthcare provider 5. Religious beliefs

Answer: 3 Explanation: This is not the greatest influence on discharge. The greatest impact on discharge is the fact that the family is homeless, all other aspects of care depend on homelessness. Page Ref: 369-372

The mother of a toddler is concerned because her child does not seem interested in eating. The child is drinking 5 to 6 cups of whole milk per day and one cup of fruit juice. When the weight-to-height percentile is calculated, the child is in the 90th to 95th percentile. What is the best advice the nurse can provide to the mother? 1. Eliminate the fruit juice from the child's diet. 2. Offer healthy snacks, presented in a creative manner, and let the child choose what he wants to eat without pressure from the parents. 3. Change from whole milk to 2 percent milk and decrease milk consumption to three to four cups per day and the fruit juice to a half cup per day, offering water if the child is still thirsty in between. 4. Make sure that the child is getting adequate opportunities for exercise, as this will increase his appetite and help lower the child's weight-to-height percentile.

Answer: 3 Explanation: Toddlers require a maximum of about 1 L of milk per day. This toddler is consuming most of his or her calories from the milk and thus is not hungry. The high fat content of the milk and the high sugar content of the fruit juice are also contributing to the child's higher weight-to-height percentile. Decreasing the amount and fat content of the milk and decreasing the intake of fruit juice will decrease calories and thus make the child hungry for other foods. The other advice is also appropriate but did not address the problem of excessive milk consumption. Page Ref: 287

A high school student calls to ask the nurse for advice on how to care for a new navel piercing. Which response by the nurse is the most appropriate? 1. "Apply warm soaks to the area for the first two days to minimize swelling." 2. "Do not move or turn the jewelry for the first 3 days." 3. "Avoid contact with another person's bodily fluids until the area is well healed." 4. "Apply lotion to the area, rubbing gently, to prevent skin from becoming dry and irritated."

Answer: 3 Explanation: Until the piercing has healed, it is a nonintact area of skin that has potential for infection, especially from contact with bodily fluids from someone else. Ice, not warm soaks, should be applied to the area for the first two days to minimize the swelling. The jewelry needs to be gently rotated several times per day to aid with healing. Lotion can provide a medium for bacteria, and rubbing at the site can cause irritation to the area. Page Ref: 383-384

A child is being discharged from the hospital after a 3-week stay following a motor vehicle accident. The mother expresses concern about caring for the child's wounds at home. She has demonstrated appropriate technique with medication administration and wound care. Which nursing diagnosis is the priority in this situation? 1. Knowledge Deficit of Home Care 2. Altered Family Processes Related to Hospitalization 3. Parental Anxiety Related to Care of the Child at Home 4. Risk for Infection Related to Presence of Healing Wounds

Answer: 3 Explanation: While all of the diagnoses might have been appropriate at some point, the current focus is the mother's anxiety about caring for the child at home. The priority of the nurse is relieving this anxiety. Page Ref: 244-245

Which aspect of an Emergency Medical Services (EMS) system is most indicative that EMS providers are prepared to provide emergency care to children? 1. Placement of small stretchers in emergency vehicles 2. Lists of hospitals in the area that treat children 3. Staff education related to assessment and treatment of children of all ages 4. Pediatric-sized equipment and supplies

Answer: 3 Explanation: While size-appropriate equipment and lists of hospitals that treat children are essential parts of an EMS system, the aspect that is most indicative that EMS providers are actually prepared to take care of children is evidence of education related to assessment and emergency treatment. Page Ref: 212

The student nurse is learning a lesson about communicable diseases and how they are spread. On a quiz the next day the nurse uses the information learned in this lesson and demonstrates learning. For a communicable disease to occur what factors must be in place? Select all that apply. 1. Antibodies 2. Toxoid 3. Pathogen 4. Transmission 5. Host

Answer: 3, 4, 5 Explanation: For a communicable disease to occur, three factors need to be in place: an infectious agent or pathogen, means of transmission, and a host. This is not a factor needed for communicable disease to occur. Page Ref: 335

A toddler recently diagnosed with a seizure disorder will be discharged home on an anticonvulsant. Which action by the mother best demonstrates understanding of how to give the medication? 1. Verbalizing how to give the medication 2. Acknowledging understanding of written instructions 3. Drawing up the medication correctly in an oral syringe and administering it to the child 4. Observing the nurse draw up the medication and administering it to the child.

Answer: 3 Explanation: 1. Verbalization of how to give the medication and acknowledging understanding of written instructions are methods that might be used, but they do not actually demonstrate understanding. Observing the nurse draw up and administer the medication may be used in the teaching process. The best way for the mother to demonstrate understanding is to actually draw up and give the medication. Page Ref: 234

The nurse is providing care to a school-age client who is admitted to the hospital after a motor vehicle accident. Which interventions are appropriate to prepare this client and family for their hospital stay? Select all that apply. 1. A hospital tour 2. A health fair brochure 3. An orientation to the unit 4. An age-appropriate explanation of procedures 5. A child life program consultation

Answer: 3, 4, 5 Explanation: Interventions that are appropriate for this client and family are those that occur as the result of an unplanned hospital admission. The nurse would orient the client and family to the unit and provide age-appropriate explanation for all procedures. It is also appropriate for the nurse to consult with the child life program. A hospital tour and a health fair brochure are appropriate interventions for a planned hospitalization. Page Ref: 230-231

A preschool-age client is seen in the clinic for a sore throat. In this child's mind, what is the most likely causative agent for the sore throat? 1. Was exposed to someone else with a sore throat. 2. Did not eat the right foods. 3. Yelled at his brother. 4. Did not take his vitamins.

Answer: 3 Explanation: Preschool-age children understand some concepts of being sick but not the cause of illness. They are likely to think that they are sick as a result of something that they have done. They will frequently view illness as punishment. A child of this age does not yet understand that he can become sick from exposure to someone else who is sick. The other two answers, while not causes of sore throat, can be factors in some illnesses but are beyond the thinking of a 4-year-old. Page Ref: 226—227

The nurse can instruct parents to expect children in which age group to begin to assume more independent responsibility for their own management of a chronic condition, such as blood-glucose monitoring, insulin administration, intermittent self-catheterization, and appropriate inhaler use? 1. Toddlers 2. Preschool-age 3. School-age 4. Adolescents

Answer: 3 Explanation: School-age children are developing a sense of industry and can begin assuming responsibility for self-care. Toddlers and preschool-age children do not have the cognitive and psychomotor skills for these tasks. Adolescents should already be well accomplished at self-care. Page Ref: 252

The school nurse is trying to prevent the spread of a flu virus through the school. Which infection-control strategies can be employed to prevent the spread of the flu virus? Select all that apply. 1. Teaching parents safe food preparation and storage 2. Withholding immunizations for children with compromised immune systems 3. Sanitizing toys, telephones, and door knobs to kill pathogens 4. Separating children with infections from children who are well 5. Teaching children to wash their hands after using the bathroom

Answer: 3, 4, 5 Explanation: To prevent the spread of communicable diseases, microorganisms must be killed or their growth controlled. Sanitizing toys and all contact surfaces, separating children with infections, and teaching children to wash their hands all control the growth and spread of microorganisms. Teaching parents safe food preparation and storage is another tool to prevent the spread of microorganisms but is not related to the flu virus. Immunizations should not be withheld from immunocompromised children; this is not an infection-control strategy. Page Ref: 363

The nurse is planning a class for school-age children on prevention of obesity through exercise. It is important to encourage the children to exercise a minimum of how many minutes a day to meet current recommendations? 1. 20 minutes 2. 30 minutes 3. 60 minutes 4. 90 minutes

Answer: 3 Explanation: The current recommendation is 60 minutes of exercise daily. Page Ref: 294-296

The nurse is caring for a child who has been sedated for a painful procedure. Which nursing activity is the priority for this child? 1. Allow parents to stay with the child. 2. Monitor pulse oximetry. 3. Assess the child's respiratory effort. 4. Place the child on a cardiac monitor.

Answer: 3 Explanation: When the child is sedated for a procedure, it is very important for the nurse to actually visualize the child and his effort of breathing. Although equipment is important and is used routinely during sedation, it does not replace the need for visual assessment. Parents may be allowed to stay with the child, but assessment of breathing effort must take priority. Page Ref: 331

The nurse is providing care to a school-age client and family. The family, which consists of two parents and 4 children, live in a one-bedroom apartment. The father recently lost his job and the mother stays at home with the children. Which community resources would most benefit this family? Select all that apply. 1. Play groups 2. Parenting programs 3. Social services programs 4. Job skills training 5. Respite care

Answer: 3, 4 Explanation: This family is currently living in a one-bedroom apartment and the sole income earner recently lost his job. This family would most benefit from social services programs for monetary assistance and job skills training which would allow the parents to learn a trade and become employed. Play groups, parenting programs, and respite care are not applicable to this family's situation. Page Ref: 210

The nurse is providing care to a pediatric client recently diagnosed with celiac disease. Which food choice indicates appropriate understanding of the material presented? 1. Pizza with milk 2. Spaghetti and meat sauce with juice 3. Hot dog on a bun with a shake 4. Fruit plate with Gatorade

Answer: 4 Explanation: A child with celiac disease needs a gluten-free diet. Included on the list are fruits, meats, rice, and vegetables, including corn. Excluded are bread, cake, doughnuts, cookies, crackers, and many processed foods that may contain hidden gluten. Therefore, the child would be allowed to have the fruit plate with Gatorade. Page Ref: 299

A school bus carrying children in grades K-12 crashed into a ravine. The critically injured children were transported by ambulance and admitted to the pediatric intensive-care unit (PICU). The nurse is concerned about calming the frightened children. Which nursing intervention is most appropriate to achieve the goal of calming the frightened children? 1. Tell the children that the physicians are competent. 2. Assure the children that the nurses are caring. 3. Explain that the PICU equipment is state of the art. 4. Call the children's parents to come into the PICU

Answer: 4 Explanation: A sense of physical and psychological security is best achieved by the presence of parents. Children at all developmental levels look first to their parents or whoever acts as their parents for safety and security. Healthcare providers, no matter how competent or caring, cannot substitute for parents. Children often neither recognize nor care about state-of-the-art equipment. Page Ref: 263

The number of serious injuries in children has doubled in the past year. Based on this information, which is the most appropriate community nursing diagnosis? 1. Noncompliance Related to Inappropriate Use of Child Safety Seats 2. Risk for Injury Related to Inadequate Use of Bicycle Helmets 3. Altered Family Processes Related to Hospitalization of an Injured Child 4. Knowledge Deficit Related to Injury Prevention in Children

Answer: 4 Explanation: All of these diagnoses might be appropriate in specific situations, but Knowledge Deficit Related to Injury Prevention in Children is the only one that is general to the problem as a whole and is therefore the most appropriate community nursing diagnosis. Page Ref: 210

The clinic nurse is working with a child with multiple disabilities. The parents have asked the nurse to help them in meeting with the school board to develop an Individualized Education Plan (IEP) and an Individualized Health Plan (IHP). Which nursing intervention is most appropriate? 1. Providing a written list of the child's medical diagnoses for the IEP meeting. 2. Offering to wait with the child while the parents attend the IEP meeting. 3. Listening to the parents' concerns and complaints about the school district. 4. Presenting verbally the child's cognitive, physical, and social skills to school officials at the IEP meeting.

Answer: 4 Explanation: As an advocate for the child and a partner with the family, the nurse attends the IEP meeting and presents the child's functional skills to develop a comprehensive IEP. A list of medical diagnoses does not accurately inform school officials about the child's skills or needs. Waiting with the child and listening to parents' concerns may be kind and empathetic but does not contribute to an action plan for the child's educational needs. Page Ref: 256-257

The mother of an infant born prematurely at 32 weeks expresses the desire to breastfeed her child. The nurse correctly responds with which statement when the mother asks how long she should breastfeed her baby? 1. "Until the child begins solid foods." 2. "Many breastfeed for 2 years." 3. "It is recommended that mothers of preterm infants breastfeed at least a month." 4. "Breast milk should be the only food for the first 6 months."

Answer: 4 Explanation: Breast milk should be the only food for the first 6 months, and should continue until 12 months even after solid foods are introduced. Page Ref: 283-284

A group of children on one hospital unit are all suffering separation anxiety. Which child is experiencing the despair stage of separation anxiety? 1. Does not cry if parents return and leave again 2. Screams and cries when parents leave 3. Appears to be happy and content with staff 4. Lies quietly in bed

Answer: 4 Explanation: Children in the despair stage appear sad, depressed, or withdrawn. A child who is lying in bed might be exhibiting any of these. Screaming and crying are components of the protest stage. The young child who appears to be happy and content with everyone is in the denial stage, as is the child who does not cry if parents return and leave again. Page Ref: 225-226

A mother of two school-age children tells the nurse that her husband has recently been deployed overseas. The mother is concerned about the children's constant interest in watching TV news coverage of military activities overseas. Which suggestion from the nurse is the most appropriate? 1. "Allow the children to watch as much television as they want. This is how they are coping with their father's absence." 2. "It will just take some time to adjust to their father's absence, then everything will return to normal." 3. "The less that you discuss this, the quicker the children will adjust to their father's absence. Try to keep them busy, and use distractions to keep their mind off of it." 4. "Spend time with your children and take cues from them about how much they want to discuss."

Answer: 4 Explanation: Constant viewing of the TV coverage of the war may increase the children's anxiety and fear for their father's safety. The mother should be aware that even though the children may appear to have adjusted, there may be delayed reactions or regressions in behavior. Children need to be able to discuss their feelings and concerns with an adult; otherwise, their emotional distress may increase. Page Ref: 371-372

A mother refuses to have her child be immunized with measles, mumps, and rubella (MMR) vaccine because she believes that letting her infant get these diseases will help him fight off other diseases later in life. Which response by the nurse is most appropriate? 1. Honor her request because she is the parent. 2. Explain that antibodies can fight many diseases. 3. Tell her that not immunizing her infant may protect pregnant women. 4. Explain that if her child contracts measles, mumps, or rubella, there could be very serious and permanent complications from these diseases.

Answer: 4 Explanation: Explaining that if her child contracts measles, mumps, or rubella, he could have very serious and permanent complications from these diseases is correct because measles, mumps, and rubella all have potentially serious sequelae, such as encephalitis, brain damage, and deafness. Honoring her request is not correct because the nurse has a professional duty to explain that the mother's belief about immunizations is erroneous and may result in harm to her infant. Explaining that antibodies can fight many diseases is not correct because the body makes antibodies that are specific to antigens of each disease. Antibodies for one disease cannot fight another disease. Telling her that not immunizing her infant may protect pregnant women is not correct because immunizing the infant with MMR vaccine will help protect pregnant women from contracting rubella by decreasing the transmission. If a pregnant woman contracts rubella, her fetus can be severely damaged with congenital rubella syndrome. Page Ref: 341

Siblings of a client in pediatric intensive care unit (PICU) are preparing to visit their brother, who was hit by a car while riding his bike. Which intervention by the nurse will assist the siblings in preparing for the visit? 1. Spend time developing a relationship with the siblings. 2. Have the parents go with the siblings when they visit. 3. Encourage the siblings to talk to a social worker before seeing their brother. 4. Explain what the siblings will hear and see when they visit.

Answer: 4 Explanation: Explaining what the siblings will hear and see when they visit will best prepare them for the visit with their brother. The other responses are good ways to help alleviate stress but won't help prepare the siblings for the visit. Page Ref: 269-270

A parent reports that her school-age child, who has had all recommended immunizations, had a mild fever one week ago and now has bright red cheeks and a lacy red maculopapular rash on the trunk and arms. Which disease process does the nurse suspect based on the parent's description? 1. Chicken pox (varicella) 2. German measles (rubella) 3. Roseola (exanthem subitum) 4. Fifth disease (erythema infectiosum)

Answer: 4 Explanation: Fifth disease manifests first with a flulike illness, followed by a red "slapped-cheek" sign. Then a lacy maculopapular erythematous rash spreads symmetrically from the trunk to the extremities, sparing the soles and palms. Varicella (chicken pox) and rubella (German measles) are unlikely if the child has had all recommended immunizations. The rash of varicella progresses from papules to vesicles to pustules. The rash of rubella is a pink maculopapular rash that begins on the face and progresses downward to the trunk and extremities. Roseola typically occurs in infants and begins abruptly with a high fever followed by a pale pink rash starting on the trunk and spreading to the face, neck, and extremities. Page Ref: 352

The nurse prepares the second diphtheria, tetanus toxoid, and acellular pertussis (DTaP) and second inactivated polio vaccine (IPV) immunization injections for an infant who is 4 months old. The nurse may also give which of immunizations during the same well-child-care appointment? 1. Var (varicella) 2. TIV (influenza) 3. MMR (measles, mumps, rubella) 4. Haemophilus influenza type B (HIB)

Answer: 4 Explanation: Haemophilus influenza type B (HIB) vaccine is given at 2, 4, 6, and 12 to 15 months of age (four doses). None of the other vaccines can be given to a 4-month-old infant. Influenza (TIV) vaccine may be given yearly to infants between 6 months and 3 years of age. Measles, mumps, and rubella (MMR) vaccine is given at 12 to 15 months and 4 to 6 years of age (two doses). Varicella (Var) is given at 12 to 18 months or any time up to 12 years for one dose; for 13 years and older two doses are given, 4 to 8 weeks apart. Page Ref: 346

Which assessment question would get the most accurate response when a nurse is assessing learning and reading skills in the early childhood years? 1. "What rewards do you use when your child does something good?" 2. "What is your child's language like now?" 3. "Does your child get along well with others?" 4. "Do you keep books for your child readily available?"

Answer: 4 Explanation: Keeping books readily available will stimulate reading skills. This is the question that will provide the most information about learning and reading skills. Language and getting along with others are more communication skills. Rewards are more closely related to discipline. Page Ref: 178

The nurse is working with a group of parents who have children with chronic conditions. Which statement by a parent would indicate a risk for a caregiver burden that could become overwhelming? 1. "My mother moved in and helped us take our quadruplets home." 2. "Our health insurance sent us a rejection letter for my child's brand-name medication, and we must fill out forms to get the generic." 3. "I chose to quit my job to be home with my child, and my husband helps in the evening when he can." 4. "I have to care for my child day and night, which leaves little time for me."

Answer: 4 Explanation: No respite time from caregiving responsibilities may lead to overwhelming caregiver burden. The family's pitching in to help indicates family support. Substituting generic for brand-name medications will not result in caregiver burden. The mother's choosing to care for the child and receiving help from the husband indicates family support. Page Ref: 258

A young school-age client is in the playroom when the respiratory therapist arrives on the pediatric unit to give the child a scheduled breathing treatment. Which action by the nurse is the most appropriate? 1. Reschedule the treatment for a later time. 2. Show the respiratory therapist to the playroom so the treatment may be performed. 3. Escort the child to his room and ask the child-life specialist to bring toys to the bedside. 4. Assist the child back to his room for the treatment but reassure the child that he may return when the procedure is completed.

Answer: 4 Explanation: Procedures should not be performed in the playroom. Scheduled respiratory treatments should be performed on time; however, the child should be allowed to return to the playroom as soon as the procedure is completed. Page Ref: 235

A young school-age client who has had a tracheostomy for several years is scheduled to begin school in the fall. The teacher is concerned about this child's being in her class and consults the school nurse. Which action by the nurse is the most appropriate? 1. Make arrangements for the child to go to a special school. 2. Ask the parents of the child to provide a caregiver during school hours. 3. Recommend that the child be home schooled. 4. Teach the teacher how to care for the child in the classroom.

Answer: 4 Explanation: Section 504 of the Rehabilitation Act of 1973 guarantees access for children with disabilities to federally funded programs, including public schools. The child may need little extra attention while in the school setting, since he has had the tracheostomy for several years. The teacher should be taught how to care for the child if needed and the signs of distress. If needed, a health aide may be assigned to the child, but this is not the responsibility of the parents. Page Ref: 211, 212

An adolescent with cystic fibrosis is intubated with an endotracheal tube. Which nursing diagnosis is most appropriate for this adolescent? 1. Potential for Imbalanced Nutrition, More Than Body Requirements Related to Inactivity 2. Anxiety Related to Leaving Chores Undone at Home 3. Potential for Fear of Future Pain Related to Medical Procedures 4. Powerlessness (Moderate) Related to Inability to Speak to or Communicate with Friends

Answer: 4 Explanation: The adolescent values communication with peers and may feel frustrated that he cannot speak to them while intubated. The adolescent is present-oriented and is unlikely to worry about household chores or future unknown procedures. The adolescent with cystic fibrosis is likely to be underweight and is unlikely to take in more calories than needed while intubated. Page Ref: 264

The nurse is reviewing the immunization record of an adolescent who will be seen later in the day. Which item in the client's history makes hepatitis B status a priority? 1. Chronic acne 2. Overuse injuries from playing varsity sports 3. Chronic asthma 4. Plans to get a tattoo

Answer: 4 Explanation: The adolescent who is most at risk in the scenario presented is the teen who is planning on getting a tattoo. Adolescents with chronic acne or asthma do not have an increased risk for hepatitis B, since transmission has nothing to do with a diagnosis of acne. Overuse of muscles while playing sports is not related to development of hepatitis B. Page Ref: 191, 192

Reducing the number of preventable childhood illnesses is a major national goal in Healthy People 2020. What will the school nurse teach families regarding immunizations in order to reach this goal? 1. A minor illness with a low-grade fever is a contraindication to receiving an immunization according to Healthy People 2020. 2. Vaccines should be given one at a time for optimum active immunity in the prevention of illness and disease. 3. Premature infants and low-birth-weight infants should receive half doses of vaccines for protection from communicable diseases. 4. It is important to maintain vaccination coverage for recommended vaccines in early childhood and to maintain them through kindergarten.

Answer: 4 Explanation: The benefits of vaccines far outweigh the risks from communicable diseases and resulting complications. A minor illness is not a contraindication to immunization. Giving vaccines one at a time will result in many missed opportunities. Half doses of vaccines should not be given routinely to premature and low-birth-weight infants. Page Ref: 341

The hospital admitting nurse is taking a history of a child's illness from the parents. The nurse concludes that the parents treated their 6-year-old child appropriately for a fever related to otitis media. Which action by the parents brought the nurse to this conclusion? 1. Used aspirin every four hours to reduce the fever 2. Alternated acetaminophen with ibuprofen every two hours 3. Put the child in a tub of cold water to reduce the fever 4. Offered generous amounts of fluids frequently

Answer: 4 Explanation: The body's need for fluids increases during a febrile illness. Aspirin has been associated with Reye syndrome and should not be given to children with a febrile illness. Alternating acetaminophen with ibuprofen every two hours may result in an overdose. Pediatric medication doses are more accurately calculated using the child's weight, not age. Putting the child in a tub of cold water will chill the child and cause shivering, a response that will increase body temperature. Page Ref: 364

What must a home-health nurse realize prior to accepting an assignment? 1. All decisions will be made by the healthcare provider. 2. The family will adapt their lifestyle to the needs of the nurse. 3. Independent decisions regarding emergency care of the child will be made by the nurse. 4. The family is in charge.

Answer: 4 Explanation: The home-health nurse must realize that the family is in charge. The nurse must be flexible and adaptable to the lifestyle of the family. The family must provide informed consent for emergency care. Page Ref: 214, 215

An infant has been NPO for surgery for 4 hours and does not have an intravenous line. The nurse receives a call from the operating room with the information that the surgery has been postponed due to an emergency. Which action by the nurse is the most appropriate? 1. Feed the infant 4 ounces of formula. 2. Reassure the parents that it will not be much longer before surgery. 3. Allow the parents to feed the infant an ounce of oral rehydration solution. 4. Call the physician to see if the infant needs to have an intravenous line started.

Answer: 4 Explanation: The infant who is NPO is at high risk for dehydration. The nurse does not know how much longer it will be before surgery. The nurse cannot independently make the decision to feed the infant. Feeding the infant could further postpone the surgery, should an operating room become available sooner than expected. It is best to keep the infant NPO and consult the physician to see if an intravenous line is needed. Page Ref: 235-238

A school-age client has been receiving morphine every two hours for postoperative pain as ordered. The medication relieves the pain for approximately 90 minutes, and then the pain returns. Which action by the nurse is the most appropriate? 1. Tell the child that pain medication cannot be administered more frequently than every two hours. 2. Reposition the child and quietly leave the room. 3. Inform the parents that the child is dependent on the medication. 4. Call the healthcare provider to see if the child's orders for pain medication can be changed.

Answer: 4 Explanation: The nurse has the responsibility of relieving the child's pain. The child has been receiving the prescribed medication on a regular basis. The healthcare provider should be called to see if the child's orders can be changed. This child might do well with patient-controlled analgesia (PCA). Oral medications such as acetaminophen and NSAIDs can be given with morphine to provide optimum pain relief. Page Ref: 319

The parents of a toddler-age child who sustained severe head trauma from falling out a second-story window are arguing in the pediatric intensive-care unit (PICU) and blaming each other for the child's accident. Which nursing diagnosis is most appropriate for this family? 1. Parental Role Conflict Related to Protecting the Child 2. Hopelessness Related to the Child's Deteriorating Condition 3. Anxiety Related to the Critical-Care-Unit Environment 4. Family Coping: Compromised, Related to the Child's Critical Injury

Answer: 4 Explanation: The parents are displaying ineffective coping behaviors as a family. Parental role conflict does not refer to the parents' argument in the PICU, but means a parent is conflicted or confused about some aspect of the parental role. Each parent may be experiencing hopelessness, frustration, and anxiety, but they are not coping well as a family unit. Page Ref: 266

The nurse must prepare parents to see their adolescent daughter in the pediatric intensive-care unit (PICU). The child arrived by life flight after experiencing multiple traumas in a car accident involving a suspected drunk driver. At this time, which statement by the nurse to the family is the most appropriate? 1. "Don't worry; everything will be okay. We will take excellent care of your child." 2. "You should press charges against the drunk driver." 3. "Your child's leg was crushed and may have to be amputated." 4. "Your child's condition is very critical; her face is swollen, and she may not look like herself."

Answer: 4 Explanation: The priority is to prepare the parents for the child's changed appearance. The nurse must not offer false reassurance nor project future stressful events. Truthful statements about the child's condition can be introduced after the parents have seen the child and grasped the situation. The nurse supports the family but remains nonjudgmental about accident details. Page Ref: 267-268

A school nurse is performing annual height and weight screening. The nurse notes that three females who are close friends each lost 15 pounds over the past year. What is the priority nursing action in this situation? 1. Call the respective parents to discuss the eating patterns of each adolescent. 2. Speak with the girls in a group to discuss the problems associated with anorexia nervosa. 3. Refer these adolescents to the school psychologist. 4. Obtain a nutritional history for each of these adolescents.

Answer: 4 Explanation: The school nurse must evaluate why these three friends have all lost 15 pounds in one year. The best way to begin this assessment is to obtain a nutritional history for each client. Speaking with the parents would not be appropriate at this time. Discussing anorexia nervosa is too extreme, as is referring the adolescents to a school psychologist without performing a complete nursing assessment. Page Ref: 185

The community-health nurse visits the child-care center. Which finding indicates the need for staff education? 1. A group of 2-year-olds are eating a snack of Cheerios. 2. Several 4-year-olds are outside playing on a slide. 3. An 18-month-old is pushing a toy truck. 4. A 2-month-old is sleeping in a crib on his stomach.

Answer: 4 Explanation: To decrease the incidence of sudden infant death syndrome (SIDS), infants should be placed on their backs to sleep. All of the other examples are developmentally appropriate activities for the specified age group. Page Ref: 210

The nurse is caring for a child who has a long leg cast. The child complains of increasing pain in the toes of the casted foot. Which initial action by the nurse is the most appropriate? 1. Call the healthcare provider to report increasing pain 2. Administer pain medication 3. Reposition the child in bed 4. Check to see if the cast is too tight

Answer: 4 Explanation: While all of the actions are appropriate, the nurse's initial action is to assess for external factors that might be causing pain. Page Ref: 324

The charge nurse is concerned with reducing the stressors of hospitalization. Which nursing intervention is most helpful in decreasing the stressors for the toddler-age client? 1. Assign the same nurse to the toddler as much as possible. 2. Let the child listen to an audiotape of the mother's voice. 3. Place a picture of the family at the bedside. 4. Encourage a parent to stay with the child.

Answer: 4 Explanation: While all of the interventions are appropriate for the hospitalized toddler, presence of a parent is most important. Separation from parents is the major stressor for the hospitalized toddler. Page Ref: 224-225

The nurse must perform a procedure on a toddler. Which technique is the most appropriate when performing the procedure? 1. Ask the mother to restrain the child during the procedure. 2. Ask the child if it is okay to start the procedure. 3. Perform the procedure in the child's hospital bed. 4. Allow the child to cry or scream.

Answer: 4 Explanation: While the toddler will need to be restrained, the parent should not be the one to do this. The nurse should avoid giving the child a choice if there is no choice. The treatment room should be utilized for the procedure so that the hospital bed remains a safe place. The child should be allowed to cry or scream during the procedure. Page Ref: 235

The nurse is working in a pediatric surgical unit. In discussing patient-controlled analgesia (PCA) in a preoperative parental meeting, which client would be a candidate for PCA? 1. Developmentally delayed 16-year-old, postoperative bone surgery 2. A 5-year-old, postoperative tonsillectomy 3. A 10-year-old who has a fractured femur and concussion from a bike accident 4. A 12-year-old, postoperative spinal fusion for scoliosis

Answer: 4 Explanation: Patient-controlled analgesia (PCA) is most appropriate in children 5 years and over. The child must be able to press the button and understand that she will receive pain medicine by pushing the button. PCA is generally prescribed for clients who will be hospitalized for at least 48 hours. Children who are developmentally delayed or have suffered head trauma are not candidates for PCA. Page Ref: 322

A mother who is bottle feeding her newborn asks to be discharged 24 hours post-delivery, because she also has twin 2-year-old children at home. When should the nurse schedule the first office visit for this newborn? 1. Within 48 hours of discharge 2. Within 1 week of discharge 3. Within 2 weeks of discharge 4. When the infant is 1-month old

Answer: 1 Explanation: 1. Newborns discharged before 48 hours old should be seen within 48 hours of discharge. Waiting 1 or 2 weeks after discharge of a 24-hour-old infant increases the chance that several common newborn conditions can go undiagnosed (e.g., jaundice, failure to gain weight). Waiting one month is too long for any infant who is discharged at 24 hours old. Page Ref: 156

A parent questions how her toddler plays with other toddlers. Which response by the nurse displays the best description of the differences in play between the toddler and the preschool-age child? 1. Toddlers play side by side, while preschool-age children play cooperatively. 2. Toddlers play house and imitate adult roles, while preschool-age children become the Mom or Dad while playing house. 3. Toddlers play cooperatively, while preschool-age children play interactive games. 4. There are no differences between toddlers and preschool-age children because both groups play cooperatively.

Answer: 1 Explanation: 1. Toddlers will play side by side with another child, but they do not interact with the child during play. Preschoolers play cooperatively with other children. Page Ref: 174

Which legal or ethical offense would be committed if a nurse tells family members the condition of a newborn baby without first consulting the parents? 1. A breach of privacy 2. Negligence 3. Malpractice 4. A breach of ethics

Answer: 1 Explanation: A breach of privacy would have been committed in this situation, because it violates the right to privacy of this family. The right to privacy is the right of a person to keep his or her person and property free from public scrutiny, including other family members. Negligence and malpractice are punishable legal offenses and are more serious. A breach of ethics would not apply to this situation. Page Ref: 9-11

A nurse is working with a pediatric client. When obtaining an accurate family assessment, which initial step is the most appropriate? 1. Establish a trusting relationship with the family. 2. Select the most relevant family-assessment tool. 3. Focus primarily upon the mother, while learning her greatest concern. 4. Observe the family in the home setting, since this step always proves indispensable.

Answer: 1 Explanation: Establishment of a trusting relationship between the family and the nurse is the essential preliminary step in obtaining an accurate family assessment. There is benefit when the tool used matches the family's strengths and resources; however, selecting the most relevant family-assessment tool is not indispensable to accuracy in the assessment. Focusing primarily upon the mother while learning her greatest concern is counterproductive and prevents the nurse from acknowledging multiple perceptions held by the family's members. Observing the family in the home setting is only recommended in some cases. Page Ref: 31

The nurse is assessing a family's effective coping strategies and ineffective defensive strategies. Which family-social-system theory is the nurse using in this assessment of the family? 1. Family-stress theory 2. Family-development theory 3. Family-systems theory 4. Family life-cycle theory

Answer: 1 Explanation: Family-stress theory indicates an array of coping strategies that effectively help reduce stress, in contrast with the defensive strategies of dysfunctional families. Family-development theory suggests developmental tasks for families in each stage. Family-systems theory looks at the relationships among and between family members and the environment. The family life cycle is not a family social system theory. Page Ref: 29, 30

The nurse is teaching a new mother developmental expectations. Which activity should the nurse expect a newborn to do within the first month of life? 1. Bring hands to eyes and mouth 2. Push up with hands, moving chest up 3. Keep hands in a relaxed position 4. Roll over from back to abdomen

Answer: 1 Explanation: Newborns at one month of age can bring hands to their eyes and mouths, move their heads from side to side when lying on their abdomens, and attempt to lift their heads only when prone. Newborn hands are kept in tight fist position, and the newborn cannot roll over until 4 months of age. Page Ref: 157

A parent says to a nurse, "How do you know when my child needs these screening tests the doctor just mentioned?" Which response by the nurse is the most appropriate? 1. "Screening tests are administered at the ages when a child is most likely to develop a condition." 2. "Screening tests are done in the newborn nursery and from these results, additional screening tests are ordered throughout the first two years of life." 3. "Screening tests are most often done when the doctor suspects something is wrong with the child." 4. "Screening tests are done at each office visit."

Answer: 1 Explanation: Screening tests administered at ages when a child is most likely to develop a condition provide a good basis for health promotion. The remaining answers all provide incorrect information to the parent. Abnormal newborn screening tests require immediate follow-up. Screening tests are done to detect the possibility of problems, not when a problem is suspected; at that point, a child needs diagnostic testing. Screening tests are not done at each office visit. Page Ref: 151

A supervisor is reviewing documentation of the nurses in the unit. Which client documentation is the most accurate and contains all the required part for a narrative entry? 1. "2/2/05 1630 Catheterized using an 8 French catheter, 45 mL clear yellow urine obtained, specimen sent to lab, squirmed and cried softly during insertion of catheter. Quiet in mother's arms following catheter removal. M. May RN" 2. "1/9/05 2 pm nasogastric tube placement confirmed and irrigated with 30 ml sterile water. Suction set at low, intermittent. Oxygen via nasal canal at 2 L/min. Nares patent, pink, and nonirritated. K. Earnst RN" 3. "4:00 tracheostomy dressing removed with dime-size stain of dry serous exudate. Site cleansed with normal saline. Dried with sterile gauze. New sterile tracheostomy sponge and trach ties applied. Respirations regular and even throughout the procedure. F. Luck RN" 4. "Feb. '05 Port-A-Cath assessed with Huber needle. Blood return present. Flushed with NaCl solution, IV gamma globulins hung and infusing at 30 cc/hr. Child smiling and playful throughout the procedure. P. Potter, RN"

Answer: 1 Explanation: The client record should include the date and time of entry, nursing care provided, assessments, an objective report of the client's physiologic response, exact quotes, and the nurse's signature and title. Page Ref: 2-4

Several children arrived at the emergency department accompanied by their fathers. Which father may legally sign emergency medical consent for treatment? 1. The divorced one from the binuclear family 2. The stepfather from the blended or reconstituted family 3. The divorced one when the single-parent mother has custody 4. The nonbiologic one from the heterosexual cohabitating family

Answer: 1 Explanation: The divorced father from the binuclear family may sign informed consent because he has equal legal rights with the mother under joint-custody arrangements. The nonbiologic stepfather from the blended or reconstituted family, the divorced biologic father when the single-parent mother has custody, and the nonbiologic father from the heterosexual cohabitating family are without legal authority to seek emergency medical care for the child. Page Ref: 20

The telephone triage nurse at a pediatric clinic knows each call is important. Which call would require attentiveness from the nurse because of an increased risk of mortality? 1. A 3-week-old infant born at 35 weeks' gestation with gastroenteritis 2. A term 2-week-old infant of American Indian descent with an upper respiratory infection 3. A postterm 4-week-old infant non-Hispanic black descent with moderate emesis after feeding 4. A 1-week-old infant born at 40 weeks' gestation with symptoms of colic

Answer: 1 Explanation: The leading causes of death in the neonatal period (birth to 28 days of age) are short gestation, low birth weight, and congenital malformations. The preterm infant experiencing gastroenteritis at 3 weeks of age is at the greatest risk for mortality; therefore, would require extra attentiveness from the registered nurse. Page Ref: 6, 7

The nurse in a pediatric acute care unit is assigned the following tasks. Which task is not appropriate for the nurse to complete? 1. Diagnose an 8-year-old with acute otitis media and prescribe an antibiotic. 2. Listen to the concerns of an adolescent about being out of school for a lengthy surgical recovery. 3. Provide information to a mother of a newly diagnosed 4-year-old diabetic about local support-group options. 4. Diagnose a 6-year-old with Diversional Activity Deficit related to placement in isolation.

Answer: 1 Explanation: The role of the pediatric nurse includes providing nursing assessment, directing nursing care interventions, and educating client and family at developmentally appropriate levels; client advocacy, case management, minimization of distress, and enhancement of coping. Advanced practice nurse practitioners perform assessment, diagnosis, and management of health conditions. Page Ref: 2-4

Which of these measures used by a nurse will help relieve parental anxiety related to the changing appetite in the toddler who is gaining weight along the 50th percentile? 1. Discussing the growth of the toddler as compared to the growth chart 2. Suggesting ways to have the toddler eat higher calorie foods 3. Instructing the mother to feed the toddler alone without any distractions such as TV or music 4. Teaching the mother to avoid disciplining the toddler within one-half hour of eating

Answer: 1 Explanation: 1. Showing the parents the growth pattern of the child as compared to the normal growth chart will help relieve parental anxiety related to eating less food during the toddler years. Toddlers who are at the 50th percentile do not need additional high-calorie foods. Toddlers eat to their personal needs and there is no reason to restrict watching TV or other environmental stimuli during meals. There is no reason to relate timing of discipline and eating. Page Ref: 170

A nurse who is the manager of an ambulatory pediatric healthcare center is planning protocols for the routine healthcare visits of the children. Children at this care center have a high incidence of obesity. At which age should the nurses at this clinic calculate the body mass index (BMI) for all pediatric clients? 1. 12 months 2. 24 months 3. 36 months 4. 4 years

Answer: 2 Explanation: The body mass index is first calculated at 2 years of age and gives information about the relationship between the height and weight of the child. With this information, the nurse would be able to develop strategies that may reduce the incidence of obesity. Page Ref: 168

While assessing the development of a 9-month-old infant, the nurse asks the mother if the child actively looks for toys when they are placed out of sight. Which developmental task is the nurse assessing this infant for? 1. Object permanence 2. Centration 3. Transductive reasoning 4. Conservation

Answer: 1 Explanation: A child who has developed object permanence has the ability to understand that even though something is out of sight, it still exists. In centration, a child focuses only on a particular aspect of a situation. Transductive reasoning happens when a child connects two events in a cause-effect relationship because they have occurred at the same time. Conservation describes when a child knows that matter is not changed when its form is altered. Page Ref: 86

A mother of a 2-year-old child becomes very anxious when the child has a temper tantrum in the medical office. Which response by the nurse is the most appropriate? 1. "What do you usually do or say during a temper tantrum?" 2. "Let's ignore this behavior; it will stop sooner or later." 3. "Pick up and cuddle your child now, please." 4. "This is definitely a temper tantrum; I know exactly what you are feeling right now."

Answer: 1 Explanation: Asking the mother to describe her usual behavior via an open-ended question will encourage the mother to talk about home management and will lead the nurse to assist the mother in making a plan of care for temper tantrums. Ignoring the behavior, instructing the mother to cuddle the child, or sympathizing with the mother ("I know exactly what you are feeling") are not effective ways to problem solve for temper tantrums. Page Ref: 149

A neonatal nurse who encourages parents to hold their baby and provides opportunities for Kangaroo Care most likely is demonstrating concern for which aspect of the infant's psychosocial development? 1. Attachment 2. Assimilation 3. Centration 4. Resilience

Answer: 1 Explanation: Attachment is a strong emotional bond between a parent and child that forms the foundation for the fulfillment of the basic need of trust in the infant. Assimilation describes the child's incorporation of new experiences, centration is the ability to consider only one aspect of a situation at a time, and resilience is the ability to maintain healthy function even under significant stress and adversity. Page Ref: 80, 83

Which of these strategies would be most effective for a teachable moment during a routine office visit for the parents of a 6-year-old child? 1. Select one topic and present a brief amount of information on the topic. 2. Review all 6-year-old anticipatory guidelines with the parents. 3. Review 7-year-old anticipatory guidelines with the parents. 4. Discuss signs of malnutrition with the parents.

Answer: 1 Explanation: Children and families often learn best when presented with small bits of information. Do not give too much information to the parents at one time; therefore, selecting one topic and presenting information is appropriate. It is not appropriate to discuss malnutrition with these parents, since nothing in the stem of the question indicates that the child has a problem with nutrition. Page Ref: 148

The nurse is assigned to a child in a spica cast for a fractured femur suffered in an automobile accident. The child's teenage brother was driving the car, which was totaled. The nurse learns that the father lost his job three weeks ago and the mother has just accepted a temporary waitress job. Which nursing diagnosis will the nurse use when planning care for this child and family? 1. Compromised Family Coping Related to the Effects of Multiple Simultaneous Stressors 2. Impaired Social Interaction (Parent and Child) Related to the Lack of Family or Respite Support 3. Interrupted Family Processes Related to Child with Significant Disability Requiring Alteration in Family Functioning 4. Risk for Caregiver Role Strain Related to Child with a Newly Acquired Disability and the Associated Financial Burden

Answer: 1 Explanation: Compromised Family Coping Related to the Effects of Multiple Simultaneous Stressors best fits the multiple crises to which this family is responding. The spica cast may require alteration in family functioning; however, fractures are generally not considered a significant long-term disability. Lack of family members and lack of respite support was not mentioned in the scenario. Page Ref: 29, 30

The nurse is counseling the parents of a 6-1/2-month-old infant. Which age-appropriate toy is most appropriate for the nurse to suggest to these parents? 1. Soft, fluid-filled ring that can be chilled in the refrigerator 2. Colorful rattle 3. Jack-in-the-box toy 4. Push-and-pull toy

Answer: 1 Explanation: Teething toys would be appropriate for this age. The rattle might be better enjoyed by a 3- to 6-month-old infant, and the jack-in-the-box and push-and-pull toys are better suited for a 9- to 12-month-old child. Page Ref: 81

A nurse is assessing an 11-month-old infant and notes that the infant's height and weight are at the 5th percentile on the growth chart. Family history reveals that the infant's two siblings are at the 50th percentile for height and at the 75th percentile for weight. Psychosocial history reveals that the parents are separated and are planning to divorce. Which of these nursing diagnoses takes priority? 1. Alteration in Growth Pattern Related to Parental Anxiety 2. Alteration in Growth Pattern Secondary to Familial Short Stature 3. Nutritional Intake: Excessive Secondary to Maternal Feeding Patterns 4. At Risk for Constitutional Growth Delay Related to Decreased Appetite

Answer: 1 Explanation: The scenario reveals parental anxiety due to marital problems. The most appropriate nursing diagnosis is alteration in growth patterns related to parental anxiety. There is no data that indicates familial short stature. Since height and weight are at the 5th percentile, there is no indication of increased nutritional intake. This infant is not at risk for constitutional growth delay. Page Ref: 162-163

A nursery nurse is planning care for the newborns currently in the newborn nursery. Which activities does the nurse plan for the first 48 hours of life? Select all that apply. 1. Monitor feeding behaviors 2. Perform a hearing screening 3. Perform a heel stick to obtain blood for the newborn screen 4. Monitor the mother as she performs the first newborn bath to remove blood and amniotic fluids 5. Administer folic-acid injection to the infant to prevent bleeding

Answer: 1, 2, 3 Explanation: The nurse should assess feeding behaviors of the infant whether the infant is breastfed or bottle-fed. A hearing screening is performed on all newborn infants prior to discharge. The newborn screen is performed prior to infant discharge from the newborn unit. The nurse, not the mother, performs the first bath to remove blood and amniotic fluids. Vitamin K is administered, not folic acid. Page Ref: 156

The clinic administrator has asked each nurse to classify the nursing activities as a beginning step of clinic reorganization. Which of these strategies can be identified as health promotion and health maintenance? Select all that apply. 1. Administration of the flu vaccine for infants from 6 months to 23 months old 2. Daily feeding schedules for infants 3. Instruction to adolescents on how to use dental floss 4. Treatment for a child with a diagnosis of acute otitis media

Answer: 1, 2, 3 Explanation: Administering flu vaccines, discussion of feeding schedules, and instructions to adolescents are all health-promotion and health-maintenance topics. Treatment of an acute ear infection (otitis media) would not be a topic for health promotion and health maintenance since it is an acute illness. Page Ref: 148

The nurse educator is teaching a group of students about the key concepts of a medical home during the developmental years of the pediatric client. Which items should the educator include in the teaching session? Select all that apply. 1. Financial accessibility 2. Consistent, ongoing care 3. Coordination of care 4. No individualization of care 5. A paternalistic view of care

Answer: 1, 2, 3 Explanation: All children need a medical home, where accessible, continuous, and coordinated health supervision is provided during the developmental years. Accessibility refers to both financial and geographic access; continuous indicates that the care is ongoing with consistent care providers; coordination refers to the need for communication among health professionals to provide for the needs of the child. Care is individualized and is not paternalistic. Page Ref: 147

The nurse in the newborn nursery is admitting a neonate. To determine the health and development of the newborn, what will the nurse include in the assessment? Select all that apply. 1. Head circumference 2. Body length 3. Weight 4. Length of pregnancy 5. Hearing screens

Answer: 1, 2, 3, 4 Explanation: 1. The nurse should assess almost all of these parameters to determine the health of the newborn. However, hearing screens are typically done after the first 12 hours after birth and are not part of newborn assessment. Page Ref: 156-157

Which nursing assessment activities should be included for the child and family at each health-supervision visit? Select all that apply. 1. Interview to obtain an updated health history 2. Performing an age-appropriate development assessment 3. Monitoring parents' ability to pay for services 4. Performing age-appropriate screening examinations 5. Physical assessment for genetic abnormalities

Answer: 1, 2, 4 Explanation: The interview, the developmental assessment, and age-appropriate screenings are all included in the nursing assessment of a child and family during each health-supervision visit. A nurse would not assess the parents' financial status at each health-supervision visit. Physical assessments for genetic abnormalities would be done based on history and physical findings, not at each routine visit. Page Ref: 150-151

The nurse is assessing a toddler's development of communication skills. The nurse recognizes that a toddler communicates in what ways? Select all that apply. 1. Expressive jargon 2. Interpersonal skills and contact with other children 3. Uses all parts of speech 4. Temper tantrums 5. Enjoys talking

Answer: 1, 2, 4, 5 Page Ref: 87

A follow-up visit for a newborn client is scheduled with the pediatric nurse practitioner 3 days after discharge. What will the nurse include in the assessment during the scheduled visit for this newborn? Select all that apply. 1. Feeding pattern 2. Jaundice 3. Length 4. Vision screen 5. Sleep pattern

Answer: 1, 2, 5 Explanation: Feeding pattern, sleep pattern, and jaundice assessment would be appropriate 3 days after discharge. It would not be necessary to do a length or vision screen at this age. Page Ref: 156

Which of the following are components of family-centered care? Select all that apply. 1. Recognizing and building on family strengths 2. Meeting the emotional, social, and developmental needs of the child and family 3. Respect all parenting practices 4. Support all cultural practices 5. Encourage parent-to-parent support

Answer: 1, 2, 5 Page Ref: 6

The nurse working with a family has observed that the older children have a large number of dental caries and plans to provide the mother with information to prevent the development of dental caries in her new infant. Which interventions will prevent the development of dental caries in the infant? Select all that apply. 1. Avoiding nursing or giving the infant a bottle at bedtime 2. Giving foods high in sugar only at breakfast time 3. Using a soft moist gauze for cleaning 4. Using a topical anesthetic daily beginning as soon as the first tooth begins to erupt

Answer: 1, 3 Explanation: The only interventions that will assist in the prevention of dental caries listed in this question are wiping the gums with a soft, moist gauze and avoiding putting the infant to bed with a bottle. Foods high in sugar should be avoided in the infant period. Topical anesthetic should not be applied daily. Page Ref: 159

The nurse is teaching a mother of a 2-month-old that she will begin to introduce certain foods to the diet between 4 and 6 months. The nurse should recommend what foods? Select all that apply. 1. Vegetables 2. Pasta 3. Rice cereal 4. Fruits 5. Soups

Answer: 1, 3, 4 Explanation: Reinforce proper introduction of new foods, to include rice cereal, vegetables, and fruits. Discuss any unusual food reactions observed. Pasta and soups are not advised at this time. Page Ref: 158

Which of these aspects of developmental health supervision should be included in each healthcare visit of young children? Select all that apply. 1. Assessment 2. Discipline 3. Education 4. Intervention 5. Toilet training

Answer: 1, 3, 4 Explanation: The main recommendations for developmental health supervision of young children include assessment, education, intervention, and care coordination. This standard framework should be used as guidelines for each healthcare visit. Discipline and toilet training, while important to the care of children, are age specific and not part of the main developmental plans. Page Ref: 168

A new parent group inquires about the stages through which their children will progress as they grow older. The nurse is discussing Piaget's developmental stages. In what order would the nurse expect the child to progress through Piaget's stages of development? 1. Sensorimotor 2. Formal operational 3. Preoperational 4. Concrete operational

Answer: 1, 3, 4, 2 Explanation: Sensorimotor (birth to 2 years), preoperational (2 to 7 years), concrete operational (7 to 11 years), formal operational (11 years to adulthood). Page Ref: 72

While in the pediatrician's office for their child's 12-month well-child exam, the parents ask the nurse for advice on age-appropriate toys for their child. Based on the child's developmental level, which types of toys would the nurse suggest? Select all that apply. 1. Soft toys that can be manipulated 2. Small toys that can pop apart and go back together 3. Jack-in-the-box toys 4. Toys with black and white patterns 5. Push-and-pull toys

Answer: 1, 3, 5 Explanation: Both gross and fine motor skills are becoming more developed, and children at this age enjoy toys that can help them refine these skills. They tend to enjoy more colorful toys at this age and are more mobile and thus have less interest in placing toys in their mouths and more interest in toys that can be manipulated. Page Ref: 67

The nurse is performing an assessment of the ecological systems of childhood. What will the nurse include when assessing mesosystems? Select all that apply. 1. Parental involvement in school 2. Local political influences 3. Libraries in the community 4. Influences of the religious community 5. Age of each family member

Answer: 1, 4 Explanation: When assessing a child's mesosystem, the nurse will assess parental involvement in school and the influences of the religious community on the child and family. Local political influences and the libraries in the community are assessed in an exosystem assessment. The age of each family member is assessed during chronosystem assessment. Page Ref: 74, 76

Match the Development surveillance questionnaire with its description. A. Denver II B. Ages and stages questionnaire C. Child development inventory D. Parents evaluation of developmental status E. Prescreening developmental questionnaire 1. Questionnaire of specific ages, 10 to 15 items in each area: fine motor, gross motor, communication, adaptive, personal, and social skills. 2. Consists of 10 questions for parent to answer in interview, based on research regarding parents' concerns. 3. Consists of 60, yes-no descriptions for three separate instruments to identify child with developmental difficulties. 4. Consists of observation of child in 4 domains; personal, social, fine-motor-adaptive, language, and gross motor. 5. Helps identify children who need Denver II assessment.

Answer: 1/B, 2/D, 3/C, 4/A, 5/E 1. Ages and stages questionnaire 2. Parents evaluation of developmental status 3. Child development inventory 4. Denver II 5. Prescreening developmental questionnaire Explanation: Ages and stages questionnaire: Questionnaire of specific ages, 10 to 15 items in each area: fine motor, gross motor, communication, adaptive, personal, and social skills. Parents evaluation of developmental status: Consists of 10 questions for parent to answer in interview, based on research regarding parents' concerns. Child development inventory: Consists of 60, yes-no descriptions for three separate instruments to identify child with developmental difficulties. Denver II: Consists of observation of child in four domains; personal, social, fine-motor-adaptive, language, and gross motor. Prescreening developmental questionnaire: Helps identify children who need Denver II assessment. These are among many developmental questionnaires available for use with children. Page Ref: 149, 150

A nurse assesses the height and weight measurements on an infant and documents these measurements at the 75th percentile. The nurse notes that the previous measurements two months ago were at the 25th percentile. Which interpretation by the nurse is the most accurate? 1. The infant is not gaining enough weight. 2. The infant has gained a significant amount of weight. 3. The previous measurements were most likely inaccurate. 4. These measurements are most likely inaccurate.

Answer: 2 Explanation: A comparison of these two sets of measurements shows that the infant has crossed two percentiles going from the 25th to the 75th percentile and therefore has gained a significant amount of weight. There is neither indication that the previous measurements are inaccurate nor that the current measurement is inaccurate. Page Ref: 157

A mother brings a child to the pediatric office for a sick visit. Which action by the nurse is the most appropriate? 1. Focus exclusively on the reported illness. 2. Review health-promotion and health-maintenance activities. 3. Ask the mother to leave the room after obtaining the history. 4. Obtain a comprehensive history, including sociodemographic data.

Answer: 2 Explanation: A nurse should use every opportunity during an office visit to review health-promotion and health-maintenance activities. Focusing exclusively on the reported illnesses ignores the opportunity to use health-promotion strategies. There is not enough data in this scenario to determine whether the mother should be asked to leave the room. There is not enough information to indicate that a comprehensive history should be taken at this visit. Page Ref: 148

A school-age client tells you that "Grandpa, Mommy, Daddy, and my brother live at my house." Which type of family will the nurse identify in the medical record based on this description? 1. Binuclear family 2. Extended family 3. Gay or lesbian family 4. Traditional nuclear family

Answer: 2 Explanation: An extended family contains a parent or a couple who share the house with their children and another adult relative. A binuclear family includes the divorced parents who have joint custody of their biologic children, while the children alternate spending varying amounts of time in the home of each parent. A gay or lesbian family is comprised of two same-sex domestic partners; they may or may not have children. The traditional nuclear family consists of an employed provider parent, a homemaking parent, and the biologic children of this union. Page Ref: 19, 20

Pediatric nurses have foundational knowledge obtained in nursing school and add specific competencies related to the pediatric client. Which would be considered an additional specific expected competency of the pediatric nurse? 1. Physical assessment 2. Anatomical and developmental differences 3. Nursing process 4. Management of healthcare conditions

Answer: 2 Explanation: Assessing anatomical and developmental differences would be a specific expected competency for the pediatric nurse that would not be learned in nursing school. Physical assessment, nursing process, and management of health conditions are all foundational knowledge learned in nursing school. Page Ref: 2-4

Which assessment would not be included with a 17-year-old's screening during a routine health supervision visit? 1. STI evaluation 2. Autism screening 3. Hemoglobin test 4. Vision screening

Answer: 2 Explanation: Autism screening would not be appropriate at this age. If autism were present, it would have presented before this age. STI evaluation, hemoglobin test, and vision screening are all appropriate for a 17-year-old. Page Ref: 151

A 7-year-old child is admitted for acute appendicitis. The parents are questioning the nurse about expectations during the child's recovery. Which information tool would be most useful in answering a parent's questions about the timing of key events? 1. Healthy People 2020 2. Clinical pathways 3. Child mortality statistics 4. National clinical practice guidelines

Answer: 2 Explanation: Clinical pathways are interdisciplinary documents provided by a hospital to suggest ideal sequencing and timing of events and interventions for specific diseases to improve efficiency of care and enhance recovery. This pathway serves as a model outlining the typical hospital stay for individuals with specified conditions. Healthy People 2020 contains objectives set by the U.S. government to improve the health and reduce the incidence of death in the twenty-first century. Child mortality statistics can be compared with those from other decades for the evaluation of achievement toward health-care goals. National clinical practice guidelines promote uniformity in care for specific disease conditions by suggesting expected outcomes from specific interventions. Page Ref: 1, 2

The nurse, talking with the parents of a toddler who is struggling with toilet training, reassures them that their child is demonstrating a typical developmental stage. According to Erikson, which developmental stage will the nurse document in the medical record for this toddler? 1. Trust versus mistrust 2. Autonomy versus shame and doubt 3. Initiative versus guilt 4. Industry versus inferiority

Answer: 2 Explanation: Erikson's stage of "autonomy versus shame and doubt" marks a period of time when the toddler is trying to gain some independence while still wanting to please adults. Page Ref: 69

A mother of an 18-month old asks the nurse whether she can begin to introduce low-fat milk like the rest of the family drinks. The nurse answers the mother based on the knowledge that low-fat milk can safely be introduced at what age? 1. 18 months 2. 24 months 3. 3 years 4. 4 years

Answer: 2 Explanation: Health promotion for the toddler includes whole milk until age 2. Age 1 is too early for low-fat milk; and it can safely be introduced before ages 3 and 4. Page Ref: 170, 171

A nurse is helping the parents of 2-year-old twins cope with the daily demands of life in an active household. Which strategy is most appropriate for the nurse to use? 1. Health maintenance 2. Health promotion 3. Health protection 4. Health supervision

Answer: 2 Explanation: In health promotion, nurses partner with families to promote family strategies in the areas of lifestyle and coping. The definition of health maintenance and health supervision makes the other answers incorrect. Health protection is another term for health maintenance. Page Ref: 146

An infant weighs 9 pounds 3 ounces at birth. The nurse plans to make a home visit to the mother and infant when the infant is 7 days old. What is the lowest acceptable weight the infant should be at this age? 1. 7 pounds 12 ounces 2. 8 pounds 2 ounces 3. 8 pounds 12 ounces 4. 9 pounds

Answer: 2 Explanation: In the first week of life, most infants lose about one-tenth of their birth weight; therefore, this infant's weight should be 8 pounds 2 ounces at 7 days of age. A weight loss to 7 pounds 12 ounces would be too much for this infant. A decline to 8 pounds 12 ounces is less than the expected one-tenth weight loss after birth, and an infant would not be expected to lose only 3 ounces during the first week of life. Page Ref: 157

A 12-year-old pediatric client is in need of surgery. Which member of the healthcare team is legally responsible for obtaining informed consent for an invasive procedure? 1. Nurse 2. Physician 3. Unit secretary 4. Social worker

Answer: 2 Explanation: Informed consent is legal preauthorization for an invasive procedure. It is the physician's legal responsibility to obtain this, because it consists of an explanation about the medical condition, a detailed description of treatment plans, the expected benefits and risks related to the proposed treatment plan, alternative treatment options, the client's questions, and the guardian's right to refuse treatment. Page Ref: 11, 12

A nurse is preparing to perform a physical assessment on a toddler. Which action is most appropriate for the nurse to take? 1. Perform the assessment from head to toe. 2. Leave intrusive procedures such as ear and eye examinations until the end. 3. Explain each part of the examination to the child before performing it. 4. Ask the mother to tell the child not to be afraid.

Answer: 2 Explanation: Intrusive procedures such as examination of the ears, throat, eye, and genital areas should be done last to decrease the anxiety of the child during the initial phases of the examination, which includes the heart and lungs. Page Ref: 168

What is the pediatric nurse's best defense against an accusation of malpractice or negligence? 1. Following the physician's written orders 2. Meeting the scope and standards of practice for pediatric nursing 3. Being a nurse practitioner or clinical nurse specialist 4. Acting on the advice of the nurse manager

Answer: 2 Explanation: Meeting the scope and standards of practice for pediatric nursing would cover the pediatric nurse against an accusation of malpractice or negligence because the standards are rigorous and cover all bases of excellent nursing practice. Following the physician's written orders or acting on the advice of the nurse manager are not enough to defend the nurse from accusations because the orders and/or advice may be wrong or unethical. Being a clinical nurse specialist or nurse practitioner does not defend the nurse against these accusations if he or she does not follow the Society of Pediatric Nurses standards of practice. Page Ref: 9-11

Parents of a preschool-age child report that they find it necessary to spank the child at least once a day. Which response should the nurse make to the parents? 1. "Spanking is one form of discipline; however, you want to be certain that you do not leave any marks on the child." 2. "Let's talk about other forms of discipline that have a more positive effect on the child." 3. "Can you try only spanking the child every other day for one week and see how that affects the child's behavior?" 4. "I think you are not parenting your child properly, so let's talk about ways to improve your parenting skills."

Answer: 2 Explanation: The behavior reported by the parent was excessive. The only response that is appropriate is to find a more positive way of influencing behavior in this age child. The nurse's response needs to reflect these feelings. To suggest spanking as an appropriate form of discipline is inappropriate, especially when the parent is describing daily spanking of the child. Page Ref: 173,174

The nurse is planning care for clients seen in a newborn clinic. Which is the priority for a newborn client during the first clinic visit? 1. Providing pamphlets to reinforce information provided at the visit 2. Assessing the newborn-family interactions 3. Modeling infant-nurturing behaviors 4. Informing the parents of the infant's gains in height and weight

Answer: 2 Explanation: The first step in the nursing process is assessment; therefore, the nurse should assess the interactions of the parents with the newborn. Providing pamphlets to help educate the parents should be done at each appropriate office visit; however, the pamphlets would be distributed after assessment of parent needs. While the nurse should be a role model for nurturing behaviors during the office visit, this would not be the first thing the nurse performs at the office visit. While parents are informed of the infant's gains in height and weight, this activity does not take priority. Page Ref: 156

The nurse recognizes that the pediatric client is from a cultural background different from that of the hospital staff. Which goal is most appropriate for this client when planning nursing care? 1. Overlook or minimize the differences that exist. 2. Facilitate the family's ability to comply with the care needed. 3. Avoid inadvertently offending the family by imposing the nurse's perspective. 4. Encourage complementary beneficial cultural practices as primary therapies.

Answer: 2 Explanation: The incorporation of the family's cultural perspective into the care plan is most likely to result in the family's ability to accept medical care and comply with the regimen prescribed. Since culture develops from social learning, attempts to ignore or minimize cultural consideration will result in mistrust, suspicion, or offenses that can have negative effects upon the health of children by reducing the resources available to promote health and prevent illness. Complementary therapy may be used later if other primary therapies prove to be ineffective. Page Ref: 3

The nurse is assessing an adolescent client whose weight is in the 5th percentile. Based on this information, which question is most appropriate for the nurse to ask the adolescent client? 1. "Do you eat the school lunches?" 2. "Do you have any concerns about your weight?" 3. "Do you eat fruits, vegetables, and drink milk?" 4. "How many meals do you eat each day?"

Answer: 2 Explanation: The only question that addresses the adolescent's weight, which is below the expected norm, is "Do you have any concerns about your weight?" Asking about school lunches, eating fruits and vegetables, and how many meals eaten each day should be used to obtain a nutritional history; however, those questions do not address the underweight status of the adolescent. Page Ref: 151

The nurse is working on parenting skills with a group of mothers. Which mother would need the fewest discipline-related suggestions? 1. Authoritarian 2. Authoritative 3. Indifferent 4. Permissive

Answer: 2 Explanation: The parental style that results in positive outcomes for the behavior and learning of its children is the authoritative style. Nurses have observed that children from homes using this parental style more frequently have personalities manifesting self-reliance, self-control, and social competence. These parents should be praised for using the preferred approach. Children in the authoritarian parenting family are denied opportunity to develop some skills in the areas of self-direction, communication, and negotiation. Under the permissive parental style, children do not learn the socially acceptable limits of behaviors. The indifferent parental style results in children who often exhibit destructive behaviors and delinquency. Page Ref: 23

A 27-month-old toddler who is in the pediatric office for a well-child visit begins to cry the moment he is placed on the examination table. The parent attempts to comfort the toddler; however, nothing is effective. Which of these actions by the nurse takes priority? 1. Instruct the father to hold the toddler down tightly to complete the examination. 2. Allow the toddler to sit on the parent's lap and begin the assessment. 3. Allow the toddler to stand on the floor until he stops crying. 4. Ask another nurse in the office to hold the toddler, because the parent is not able to control the toddler's behavior.

Answer: 2 Explanation: Toddlers are most comfortable when sitting with the parents. Much of the examination can be completed in this way. Allowing the toddler to stand on the floor is inappropriate. A nurse can assist if the parent is unable to hold the child during the examination of the throat and ears to prevent injury from movement. Page Ref: 168, 169

The nurse is preparing to perform a hearing screening on a 6-year-old child. The nurse knows this screening is what level of prevention? 1. Primary prevention 2. Secondary prevention 3. Tertiary prevention 4. Quaternary prevention

Answer: 2 Page Ref: 146

In the pediatric well-child clinic, the nurse explains the reason for an immunization series to the child's mother. This action represents which item? 1. Health assessment 2. Health promotion 3. Health maintenance 4. Health screening

Answer: 2 Explanation: The explanation to the mother by the nurse provides an understanding of the immunization series to the mother and enables the mother to make an intelligent choice. While administering immunizations is considered health maintenance, the activity described in the question is clearly health promotion. A health assessment would be completed to determine what immunizations are needed. Health maintenance is the actual administration of the immunization and health screening involves looking at the immunization record to determine which immunizations are needed. Page Ref: 146

There are several tools that help with obtaining a cultural assessment of a client and his family. Which tool would be appropriate to gather 12 major concepts of cultural assessment? 1. Sunrise enabler 2. Model for cultural competence 3. Transcultural assessment model 4. Health traditions model

Answer: 2 Explanation: The sunrise enabler examines influences on care and culture. The model for cultural competence will gather information on 12 major concepts. The transcultural assessment model is based on 6 phenomena. The health traditions model is predicated on holistic health.

The nurse is counseling the parents of a 13-year-old regarding the behaviors they may encounter after telling the child about their plans to divorce. Which behaviors could the child demonstrate? Select all that apply. 1. Sorrow 2. Skipping school 3. Risk-taking 4. Withdraw from friends and activities 5. Temper tantrums

Answer: 2, 3 Page Ref: 25, 26

A nurse is working with pediatric clients in a research facility. The nurse recognizes that federal guidelines are in place that delineate which pediatrics clients must give assent for participation in research trials. Based upon the client's age, the nurse would seek assent from which children? Select all that apply. 1. The precocious 4-year-old commencing as a cystic fibrosis research-study participant. 2. The 7-year-old leukemia client electing to receive a newly developed medication, now being researched. 3. The 10-year-old commencing in an investigative study for clients with precocious puberty. 4. The 13-year-old client commencing participation in a research program for Attention Deficit Hyperactivity Disorder (ADHD) treatments.

Answer: 2, 3, 4 Explanation: Federal guidelines mandate that research participants 7 years old and older must receive developmentally appropriate information about healthcare procedures and treatments and give assent. Page Ref: 11, 12

The nurse is planning care for an adolescent client who will be hospitalized for several weeks following a traumatic brain injury. Which interventions will enhance family-centered care for this client and family? Select all that apply. 1. Making all ADL decisions for the adolescent and family 2. Asking the adolescent what foods to include during meal time 3. Allowing the family time to pray each day with the adolescent 4. Encouraging the adolescent's friends to visit during visiting hours 5. Leaving all questions for the healthcare provider

Answer: 2, 3, 4 Page Ref: 18, 19

The nurse is planning care for a school-age client and family who have expressed wanting to use complementary and alternative modalities (CAM) in the treatment plan. Which interventions can the nurse safely implement into the plan of care? Select all that apply. 1. Substituting an herbal remedy for a prescribed medication 2. Encouraging the parents to share which modalities they would like to implement 3. Educating on the benefits and risks for each modality 4. Using essential oils to decrease nausea 5. Discouraging the use of faith-based therapies

Answer: 2, 3, 4 Explanation: Appropriate interventions for this client and family include encouraging the parents to share which modalities they want to implement, educating about the risks and benefits of each modality, and using modalities that are safe, such the use of essential oils to decrease nausea. An herbal remedy should not be substituted for a prescribed medication, but can be used if deemed safe with the prescribed medication. Discouraging the use of faith-based therapies does not support the client and family who want to use CAM in the treatment plan. Page Ref: 38-40

Which of these developmental milestones should the nurse expect to find in children who are between 2 and 3 years old? Select all that apply. 1. Always feeds self 2. Scribbles and draws on paper 3. Kicks a ball 4. Throws ball overhand 5. Goes up and down stairs

Answer: 2, 3, 5 Explanation: Children between the ages of 2 and 3 years can scribble and draw on paper, kick a ball, and go up and down the stairs. Children who are between the ages of 3 and 4 years can feed themselves. Children between the ages of 4 and 5 years can throw a ball overhand. Page Ref: 171

A nurse and the family of an 8-year-old with acute renal failure are reviewing family strengths helpful in managing stressors. Which family strengths should the nurse recommend this family utilize? Select all that apply. 1. Meeting member needs 2. Support by extended family 3. Effective communication 4. Receiving and giving love 5. Prior life experiences

Answer: 2, 3, 5 Page Ref: 31

The nurse is teaching the mothers of three-month-olds about oral health. Which of the following should the nurse include? Select all that apply. 1. Include iron vitamins once a day. 2. Avoid breastfeeding or drinking from a bottle when sleeping. 3. Allow to drink from a bottle at will during the day. 4. Cleanse gums 1 to 2 times a day. 5. Put baby to bed with a bottle of 2 percent milk only.

Answer: 2, 4 Explanation: The parents should wipe the infant's gums with soft moist gauze once or twice daily. Families are also cautioned to avoid having the infant breastfeed when sleeping, to avoid use of bottles in bed, and not to allow the infant to drink at will from a bottle during the day. These practices are linked to early childhood caries and can lead to tooth decay. Page Ref: 159

A 12-year-old child is admitted to the unit for a surgical procedure. The child is accompanied by two parents and a younger sibling. What is the level of involvement in treatment decision making for this child? 1. Emancipated minor 2. Mature minor 3. Assent 4. None

Answer: 3 Explanation: Assent requires the ability to generally understand what procedure and treatments are planned, to understand what participation is required, and to make a statement of agreement or disagreement with the plan. Usually, in Piaget's stage of formal operations, 11- to 13-year-olds should be able to problem solve using abstract concepts and are able to give valid assent when parents sign the informed consent. An emancipated minor is a self-supporting adolescent who is not subject to the control of a parent or guardian. A mature minor is a 14- or 15-year-old whom the state law designates as being able to understand medical risks and who is thus permitted to give informed consent for treatment. Page Ref: 11, 12

The nurse is working on parenting skills with a mother of three children. The nurse demonstrates a strategy that uses reward to increase positive behavior. Which strategy will the nurse document in the medical record based on this description? 1. Time out 2. Reasoning 3. Behavior modification 4. Experiencing consequences of misbehavior

Answer: 3 Explanation: Behavior modification identifies and gives rewards for desired behaviors. Time out and experiencing consequences of misbehavior show the child that unacceptable behavior brings undesirable outcomes. Reasoning attempts to use explanation to end misbehavior. Page Ref: 24

The nurse is assessing a group of children attending summer camp. The nurse will expect which children to most likely have problems perceiving a sense of belonging? 1. Children whose parents divorced recently 2. Children who gained a stepparent recently 3. Children recently placed into foster care 4. Children adopted as infants

Answer: 3 Explanation: Children in foster care are more likely to have problems perceiving a sense of belonging. Children whose parents divorce often fear abandonment. Children who gain a stepparent may have problems trusting the new parent. Infants who are adopted at birth can have minimal problems with acceptance when parents follow preadoption counseling about disclosure. Page Ref: 27

Despite the availability of Children's Health Insurance Programs (CHIP), many eligible children are not enrolled. Which nursing intervention would be the most appropriate to help children become enrolled in CHIP? 1. Assess details of the family's income and expenditures 2. Case management to limit costly, unnecessary duplication of services 3. Advocate for the child by encouraging the family to investigate SCHIP eligibility 4. Educate the family about the need for keeping regular well-child-visit appointments

Answer: 3 Explanation: In the role of an advocate, a nurse will advance the interests of another; by suggesting the family investigate its CHIP eligibility, the nurse is directing their action toward the child's best interest. Financial assessment is more commonly the function of a social worker. The case-management activity mentioned will not provide a source of funding nor will the educational effort described. Page Ref: 2-4

A nurse asks the mother of a 4-month-old infant to undress the infant. The nurse observes the mother taking off several layers of clothing and knows that the outdoor temperature is 70 degrees Fahrenheit. Which statement by the nurse is most appropriate in this situation? 1. "My, you are dressing your infant warmly today." 2. "Did you think it was cold when you left your home this morning?" 3. "I see that you have many layers of clothing on your baby. This may cause your baby's temperature to rise." 4. "When you leave the office, only put one layer of clothing on your baby."

Answer: 3 Explanation: In this scenario, the mother has overdressed the infant. The nurse needs to gently inform the mother of this problem and to provide information to the mother on why it is a problem. Just making a statement on how warmly the child is dressed will not accomplish this goal or just telling the mother to only put one layer of clothing on the child does not provide a rationale for the mother to make a better decision the next time, so this statement also is not helpful to the mother. Page Ref: 157

A mother asks which developmental milestones she can expect when her baby is 6 months old. Which response by the nurse is the most appropriate? 1. Lifts head momentarily when prone 2. Has well-developed pincer grasp 3. Transfers objects from one hand to the other 4. Rolls from front to back

Answer: 3 Explanation: Lifting head when prone is a milestone at 1 month. A well-developed pincer grasp is a milestone at 12 months. Transferring objects from one hand to the other is a milestone at 6 months. Rolling from front to back is a milestone at 4 months. Page Ref: 157-158

The nurse in the pediatric clinic observes a parental lack of warmth and interest toward the child. Which parental style will the nurse most likely document in this situation? 1. Authoritarian 2. Authoritative 3. Indifferent 4. Permissive

Answer: 3 Explanation: Parents displaying the indifferent parental style fail to demonstrate consistent warmth and interest in their children. Parents who favor the authoritarian style may exhibit a punitive attitude toward the child who is misbehaving but are not disinterested. Parents employing the authoritative style and the permissive style have children who report that "my parent loves me and shows affection regularly." Page Ref: 23

An adolescent client with cystic fibrosis suddenly becomes noncompliant with the medication regime. Which intervention by the nurse will most likely improve compliance for this client? 1. Give the child a computer-animated game that presents information on the management of cystic fibrosis. 2. Arrange for the physician to sit down and talk to the child about the risks related to noncompliance with medications. 3. Set up a meeting with some older teens with cystic fibrosis who have been managing their disease effectively. 4. Discuss with the child's parents the privileges that can be taken away, such as cell phone, if compliance fails to improve.

Answer: 3 Explanation: Providing an adolescent with positive role models who are in his peer group is the intervention most likely to improve compliance. Interest in games may begin to wane, adults' opinions may be viewed negatively and challenged, and threatening punishment may further incite rebellion. Page Ref: 96

Which nursing intervention is most appropriate when providing education to the pediatric client and family? 1. Giving primary care for high-risk children who are in hospital settings 2. Giving primary care for healthy children 3. Working toward the goal of informed choices with the family 4. Obtaining a physician consultation for any technical procedures at delivery

Answer: 3 Explanation: The educator works with the family toward the goal of making informed choices through education and explanation. Page Ref: 3

The community health nurse is assessing several families for various strengths and needs in regard to after-school and backup childcare arrangements. Which family type will benefit the most from this assessment and subsequent interventions? 1. The binuclear family 2. The extended family 3. The single-parent family 4. The traditional nuclear family

Answer: 3 Explanation: The single-parent family most typically lacks social, emotional, and financial resources. Nursing considerations for such families should include referrals to options that will enable the parent to fulfill work commitments while providing the child with access to resources that can support the child's growth and development. The binuclear family, the extended family, and the traditional nuclear family generally have at least two adults who can share in the care and the nurturing of its children. Page Ref: 19, 20

At a routine healthcare visit, a nurse measures a toddler and plots the height and weight on the growth charts. The nurse documents that the toddler is above the 95th percentile for weight and is at the 5th percentile for height. How should the nurse interpret these data? 1. The toddler is proportionate for the age. 2. The toddler needs to eat more at each feeding. 3. The height and weight are disproportionate, and the toddler needs further evaluation. 4. The family is most likely short.

Answer: 3 Explanation: Usually height and weight are at approximately the same percentile. When the weight of a child is found to be at the 95th percentile, the child's height is also greater than the 50th percentile. The height and weight for the child described in this question are a concern, and the child may need further endocrine testing. Page Ref: 168

The parents of an 8-year-old state that their son seems very interested in trying new activities. When the parents ask for suggested activities for this age child, the nurse recommends scouts as an activity that will foster growth and development. In which stage of Erikson's "psychosocial stages of development" is this child? 1. Trust versus mistrust 2. Initiative versus guilt 3. Industry versus inferiority 4. Identity versus role confusion

Answer: 3 Page Ref: 69

The parents of a 1-year-old infant are concerned that this baby seems more shy and scared of new situations than their other child and ask the nurse if this is normal. The nurse knows that the infant is exhibiting a characteristic of the "slow-to-warm-up." Which statement to the parents is most appropriate by the nurse? 1. "Your infant is showing a regularity in patterns of eating." 2. "Your infant displays a predominately negative mood." 3. "Your infant initially reacts to new situations by withdrawing." 4. "Your infant has intense reactions to the environment."

Answer: 3 Explanation: "Slow-to-warm-up" children adapt slowly to new situations and initially will withdraw. Showing regularity in patterns of eating is a characteristic of an "easy" child, and displaying a predominately negative mood and commonly having intense reactions to the environment are characteristics of "difficult" children. Page Ref: 77, 78

Injury prevention is an important aspect of parent teaching. Which injury prevention strategy would reduce the risk of suffocation? 1. Measure crib slat spacing at 2-3/8 inches or less. 2. Never leave an infant alone in a bath. 3. Position the infant on her back to sleep. 4. Use only approved restraint systems.

Answer: 3 Explanation: Measuring crib slats will reduce strangulation. Not leaving an infant alone in a bath will reduce drowning. Positioning an infant on her back will reduce suffocation. Using approved restraint systems will reduce motor vehicle injury. Page Ref: 162

Two 3-year-olds are playing in a hospital playroom together. One is working on a puzzle while the other is stacking blocks. Which type of play are these children exhibiting? 1. Cooperative play 2. Associative play 3. Parallel play 4. Solitary play

Answer: 3 Explanation: Parallel play describes when two or more children play together, each engaging in their own activities. Cooperative play happens when children demonstrate the ability to cooperate with others and play a part in order to contribute to a unified whole. Associative play is characterized by children interacting in groups and participating in similar activities. In solitary play, a child plays alone. Page Ref: 86

Cultures have many different childrearing practices. Which culture is known to value the male child more than the female child, and often teaches children to avoid displaying emotion? 1. Mexican 2. Amish 3. Chinese 4. Navajo

Answer: 3 Explanation: The Chinese culture values the male child more than the female child, and often teaches children to avoid showing emotion. The other cultures do not have this component. Page Ref: 35

A nurse observes the parent/child interaction during the 4-year-old well-child checkup and notes that the parent speaks harshly to the child and uses negative remarks when speaking with the nurse. Which statement by the nurse would be most beneficial? 1. "Perhaps you should leave the room so that I can speak with your child privately." 2. "I am going to refer you for counseling since your interactions with your child seem so negative." 3. "Let's talk privately. Let's discuss the way you speak with your child and possible ways to be more positive." 4. Addressing the child, the nurse says, "Are you unhappy when Mommy talks to you like this?"

Answer: 3 Explanation: The best approach to this encounter would be for the nurse to discuss concerns with the parent privately, since the nurse wants to help the parent develop a good relationship with the child. The child should not be a part of this conversation. Because the child is only 4 years old, it would be difficult to ask the parent to leave the room. If the nurse also wants to speak alone with the child, the nurse perhaps would escort the child to another area and speak briefly with the child. Referring to counseling without a discussion with the parent is not appropriate. The nurse should not ask the child if she is "unhappy" with the parent. Page Ref: 174, 175

A mother of a school-age client who recently had surgery for the removal of tonsils and adenoids complains that the child has begun sucking his thumb again. Which coping mechanisms is the child using to cope with the surgery and hospitalization? 1. Repression 2. Rationalization 3. Regression 4. Fantasy

Answer: 3 Explanation: The correct answer is regression, which is a return to an earlier behavior. Repression is the involuntary forgetting of uncomfortable situations, rationalization is an attempt to make unacceptable feelings acceptable, and fantasy is a creation of the mind to help deal with an unacceptable fear. Page Ref: 68

Which statement by the nurse is most appropriate prior to giving an intramuscular injection to a 2-1/2-year-old child? 1. "We will give you your shot when your mommy comes back." 2. "This is medicine that will make you better. First we will hold your leg, then I will wipe it off with this magic cloth that kills the germs on your leg right here, then I will hold the needle like this and say 'one, two, three . . . go' and give you your shot. Are you ready?" 3. "It is all right to cry, I know that this hurts. After we are done you can go to the box and pick out your favorite sticker." 4. "This is a magic sword that will give you your medicine and make you all better."

Answer: 3 Explanation: The most appropriate response would be to acknowledge the child's feelings and give her something to look forward to (picking out a sticker). Waiting for the mother to come back would be inappropriate because toddlers do not have an understanding of time. Giving elaborate descriptions and using colorful language are inappropriate. The instructions should not end with a "are you ready" statement because the toddler will say no. You also don't want to frighten and/or confuse the child by using statements such as use of a magic sword. Page Ref: 74

A nurse in the outpatient pediatric clinic is reviewing the records of a preschool-age child and notes that because the parents often miss routine healthcare visits the child has not received the second measles, mumps, and rubella (MMR) vaccine. Which action by the nurse is most appropriate in this situation? 1. Speak firmly with the parents about the importance of being compliant. 2. Notify the physician that the child's immunizations are no longer up to date. 3. Call the parents and encourage them to bring the child for recommended care. 4. Plan to discuss the principles of health supervision at the next scheduled visit.

Answer: 3 Explanation: The nurse in the pediatric healthcare setting is responsible for reviewing the health supervision of the child. Partnering with the parents and encouraging the parents to follow health-supervision guidelines are the best strategies to use. Speaking firmly with the parents about compliance will alienate the parents at this time. A discussion of the principles of health supervision without an intervention at this visit would mean a delay in needed healthcare for the child in this example. Discussing with the physician that the immunizations are not up to date is not necessary in an outpatient clinic. Immunizations are given per schedule. Page Ref: 147

The nurse educator is presenting a lecture about risks to developmental progression. Which items will the educator include in the lecture? Select all that apply. 1. Family support 2. Access to the Internet 3. Recent loss of employment 4. Terminal illness of a family member 5. Hazards within the home environment

Answer: 3, 4, 5 Explanation: Risk factors that can inhibit developmental progression include financial problems, stresses and worries, family and job instability, neighborhood and home hazards, and lack of resources. Family support and access to the Internet are both considered protective factors. Page Ref: 94-96

The nurse is assessing an infant client and parents during a routine health supervision visit at 2 months of age. Which items will the nurse assess to determine if the infant's mental health needs are being addressed? Select all that apply. 1. Immunization record 2. Newborn screen results 3. Temperament during the visit 4. Feeding schedule 5. Sleep-wake patterns

Answer: 3, 4, 5 Explanation: When addressing mental health issues, the nurse would assess the infant's temperament during the visit, feeding schedule, and sleep-wake patterns. The infant's mental health is related to early experiences, inborn characteristics such as temperament and resilience, and relationships with caregivers. The first year of life provides many opportunities for the infant to develop positive mental health; interventions during this important period can enhance the child's future mental status. The immunization record and the newborn screen results will not provide the needed information for the nurse in terms of whether the infant's mental health needs are being addressed. Page Ref: 159-160

The nurse of an outpatient clinic is sitting with the parents while their adolescent goes for a test. The parents are complaining about their child's behavior. Which statement by the nurse fosters family-centered communication? 1. "I agree with you; discipline is an important part of parenting." 2. "I know just how you feel. I had the same experience with my children." 3. "You are so right. Adolescents function in the "me-first" mode all the time." 4. "Tell me what concerns you about your child's behavior."

Answer: 4 Explanation: 1. Using an open-ended question allows the parents to discuss a family concern. All the other questions or statements are blocking statements and would not foster family-centered communication. Page Ref: 151

A child is being prepared for an invasive procedure. The mother of the child has legal custody but is not present. After details of the procedure are explained, who can provide legal consent on behalf of a minor child for treatment? 1. The divorced parent without custody 2. A cohabitating boyfriend of the child's mother 3. A grandparent who lives in the home with the child 4. A babysitter with written proxy

Answer: 4 Explanation: A parent may grant proxy consent in writing to another adult so that children are not denied necessary healthcare. In the case of divorced parents, the parent with custody may be the only parent allowed by some states to give informed consent. Residence in the same household with a child does not authorize an adult to sign consent for treatment. Page Ref: 11, 12

Which nursing role is not directly involved when providing family-centered approach to the pediatric population? 1. Advocacy 2. Case management 3. Patient education 4. Researcher

Answer: 4 Explanation: A researcher is not involved in the family-centered approach to patient care of children and their families. Advocacy, case management, and patient education are all roles directly involved in the care of children and their families. Page Ref: 4

While teaching parents of a newborn about normal growth and development, which statement is most appropriate for the nurse to include in the session? 1. Weight should triple by 6 months of age. 2. Weight should double by 1 year of age. 3. Weight should double by 4 months of age. 4. Weight should triple by 1 year of age.

Answer: 4 Explanation: An infant should triple its birth weight by 1 year of age. The other answers are not appropriate weight gains. Page Ref: 158

The nurse is providing anticipatory guidance instructions to the parents of a newborn. Which instruction should the nurse give as a strategy for illness/disease prevention? 1. Don't allow visitors for the first month 2. Smoke outside only 3. Take the newborn to weekly child-stimulation classes 4. SIDS risk-reduction measures

Answer: 4 Explanation: Several disease-prevention strategies are used during anticipatory guidance for the parents of newborns. Not allowing visitors is unreasonable but screening for illness is appropriate. Smoking outside will not prevent disease. Attending weekly stimulation classes is not a disease prevention strategy. SIDS risk-reduction measures can reduce the risk of sudden infant death syndrome. Page Ref: 157

A nurse is discussing health promotion activities with parents of a 4-year-old client. What health-promotion activity is most appropriate for this family? 1. Make arrangements to tour the kindergarten in which the child will enroll next year. 2. Plan a "movie afternoon" with the child's big brother. 3. Maintain appropriate immunizations. 4. Teach the child the proper method for brushing the teeth

Answer: 4 Explanation: Teaching proper oral hygiene through proper teeth brushing is a health-promotion activity. Touring the kindergarten might alleviate anxiety, but is not health promotion. A movie afternoon with the big brother is sedentary, and also not a health-promotion activity. Maintaining immunizations is a health-maintenance, not health-promotion, activity. Page Ref: 149

A new pediatric hospital will open soon. While planning nursing care, the hospital administration is considering two models of providing healthcare: family-focused care and family-centered care. Which action best demonstrates family-centered care? 1. Telling the family what must be done for the family's health 2. Assuming the role of an expert professional to direct the healthcare 3. Intervening for the child and family as a unit 4. Conferring with the family in deciding which healthcare option will be chosen

Answer: 4 Explanation: The benefit of employing the family-centered-care philosophy is that the priorities and needs as seen by the family are addressed as a partnership between a family and a nurse develops. In family-focused care, the healthcare worker assumes the role of professional expert while missing the multiple contributions the family brings to the healthcare meeting. Page Ref: 18, 19

The nurse is performing an assessment of a child's biologic family history. Which situation would necessitate the nurse's asking the mother for information should use the term "child's father" instead of "your husband"? 1. Traditional nuclear family 2. Traditional extended family 3. Two-income nuclear family 4. Cohabitating informal stepfamily

Answer: 4 Explanation: The mother from the cohabitating informal stepfamily does not have a husband; the nurse should be asking about the "child's father." In the traditional nuclear family, the traditional extended family, and the two-income nuclear family, the child's father is the same person as the mother's husband. Page Ref: 19, 20

A nurse says to the mother of a 6-month-old infant, "Does the baby sit without assistance, and is the baby crawling?" Which process is the nurse using in this interaction? 1. Health promotion 2. Health maintenance 3. Disease surveillance 4. Developmental surveillance

Answer: 4 Explanation: The question asked by the nurse is seeking information about developmental milestones; therefore, the nurse is involved in developmental surveillance. While health-promotion and health-maintenance activities are related to developmental surveillance, this question is looking specifically at the milestones; therefore, the answers "health promotion" and "health maintenance" are incorrect. The questions asked in the stem are not classified as disease-surveillance questions. Page Ref: 149

A pediatric nurse who is employed in a busy ambulatory clinic setting is informed by the nurse manager that average nursing time allocated for each child and family is being reduced to 10 minutes to more efficiently manage the clinic. The nursing activities must include a nursing assessment and discussion on anticipatory guidance. Which of these strategies should the nurse utilize in the plan of care delivery? 1. Attempt to complete the assessment and education in 10 minutes, but extend the time whenever the nurse deems necessary. 2. Plan to do the anticipatory guidance first since either the nurse practitioner or the physician can perform the assessment of the child. 3. Encourage the parent to ask for specific time to talk with the nurse privately at each office visit. 4. Focus anticipatory guidance strategies on topics that the parent or child have expressed as an area of interest.

Answer: 4 Explanation: With limited time for each visit, the nurse should focus on anticipatory guidance strategies that will most benefit the parent and child during that office visit. Page Ref: 148

While being comforted in the emergency department, a young school-age sibling of a pediatric trauma victim blurts out to the nurse, "It's my fault! When we were fighting yesterday, I told him I wished he was dead!" Which response is most appropriate by the nurse? 1. Asking the child if she would like to sit down and drink some water 2. Sitting the child down in an empty room with markers and paper so that she can draw a picture 3. Calmly discussing the catheters, tubes, and equipment that the patient requires and explaining to the sibling why the patient needs them 4. Reassuring the child that it is normal to get angry and say things that we do not mean but that we have no control over whether or not an accident happens

Answer: 4 Explanation: "Magical thinking" is the belief that events occur because of one's thoughts or actions, and the most therapeutic way to respond to this is to correct any misconceptions that children may have and reassure them that they are not to blame for any accidents or illness. Page Ref: 89, 90

While trying to inform a young school-age client about what will occur during an upcoming CT scan, the nurse notices that the child is engaged in a collective monologue, talking about a new puppy. Which response by the nurse is the most appropriate in this situation? 1. "Please stop talking about your puppy. I need to tell you about your CT scan." 2. Ignore the child's responses and continue discussing the procedure. 3. "I'll come back when you are ready to talk with me more about your CT scan." 4. "You must be so excited to have a new puppy! They are so much fun. Now, let me tell you again about going downstairs in a wheelchair to a special room."

Answer: 4 Explanation: When a child becomes engaged in a collective monologue, it is best to respond to the content of his or her conversation and then attempt to reinsert facts about the content that needs to be covered. Page Ref: 90

A three-week-old infant is returned post-pyloromyotomy three hours ago. The father is refusing pain medication for the infant and states, "The baby is hungry. Can I give the baby a bottle?" How should the nurse best advocate for the infant? Select all that apply. 1. Call the physician to ask if the child can feed yet. 2. The FLACC scale rating is 8 out of 10; try swaddling and rocking the infant. 3. Ask the parent to obtain a FLACC scale rating and let the nurse know what rating they get. 4. Educate the parent about the surgery and why the infant should not have anything by mouth. 5. Inform the parent about the meaning of the pain scale and the need for pain medication.

Answer: 4, 5 Page Ref: 3


Set pelajaran terkait

földrajz- a nemzetgazdaságtól a globális világgazdaságig

View Set

Portuguese Grammar Test - revision

View Set

Chapter 14 Genetics Study Guide Questions

View Set

What is unique about The DNA of a prokaryote

View Set

Studies Weekly 18 - The Germanic Tribes

View Set